Формула последовательного соединения конденсаторов: Схемы соединения конденсаторов — расчет емкости

Содержание

Формула расчета последовательного соединения конденсаторов. Соединение конденсаторов параллельное соединение конденсаторов

Вопрос о том, как соединить конденсаторы может возникнуть у любого человека, интересующегося электроникой и пайкой . Чаще всего, необходимость в этом возникает в случаях отсутствия под рукой устройства подходящего номинала при сборке или ремонте какого-либо прибора.

К примеру, человеку нужно отремонтировать устройство, заменив в нем электролитический конденсатор ёмкостью 1000 микрофарад или больше, на руках подходящие по номиналу детали отсутствуют, но есть несколько изделий с меньшими параметрами. В этом случае есть три варианта выхода из сложившейся ситуации:

  1. Поставить вместо конденсатора на 1000 микрофарад устройство с меньшим номиналом.
  2. Поехать в ближайший магазин или радио-рынок для покупки подходящего варианта.
  3. Соединить несколько элементов вместе для получения необходимой ёмкости.

От установки радиоэлемента меньшего номинала лучше отказаться, так как подобные эксперименты не всегда заканчиваются успешно. Можно съездить на рынок или в магазин, но это требует немало времени. Потому в сложившейся ситуации чаще соединяют несколько конденсаторов и получают необходимую емкость.

Параллельное соединение конденсаторов

Параллельная схема подключения конденсаторов предполагает соединение в две группы всех обкладок приборов. В одну группу соединяются первые выводы, а в другую группу – вторые выводы. На рисунке ниже представлен пример.

Конденсаторы, соединенные параллельно между собой, подключаются к одному источнику напряжения, поэтому на них существует две точки напряжения или разности потенциалов . Следует учитывать, что на всех выводах подключенных параллельно конденсаторов напряжение будет иметь одинаковую величину.

Параллельная схема образует из элементов единую ёмкость, величина которой равняется сумме ёмкостей всех подключенных в группу конденсаторов. При этом через конденсаторы в процессе работы устройства будет протекать ток разной величины. Параметры проходящего через изделия тока зависят от индивидуальной ёмкости устройства. Чем выше ёмкость, тем больший по величине ток пройдет через него. Формула, характеризующее параллельное соединение, имеет следующий вид:

Параллельная схема чаще всего используется в быту, она позволяет собрать необходимую ёмкость из любого числа отдельных, различных по номиналу элементов.

Последовательное соединение конденсаторов

Схема последовательного подключения представляет собой цепочку, в которой первая обкладка конденсатора соединяется со второй обкладкой предыдущего устройства, а вторая обкладка – с первой обкладкой следующего прибора. Первый вывод первого конденсатора и второй вывод последней детали в цепи соединяются с источником электрического тока, благодаря чему между ними осуществляется перераспределение электрических зарядов. Все промежуточные обкладки имеют одинаковые по величине заряды, чередующиеся по знаку.

На рисунке ниже представлен пример последовательного подключения.

Через соединенные в группу конденсаторы протекает ток одинаковой величины. Общая мощность ограничивается площадью обкладок устройства с наименьшим номиналом, так как после зарядки наименьшего по ёмкости устройства, вся цепь перестанет пропускать ток.

Несмотря на явные недостатки, данный способ обеспечивает увеличение изоляции между отдельными обкладками до суммы расстояний между выводами на всех последовательно соединенных конденсаторах. То есть, при последовательном соединении двух элементов с рабочим напряжением 200 В, изоляция между их выводами сможет выдерживать напряжение до 1000 В. Ёмкость по формуле:

Данный способ позволяет получить эквивалент меньшего по ёмкости конденсатора в группе, способной работать при высоких напряжениях. Всего этого можно достичь путем покупки одного единственного элемента подходящего номинала, потому на практике последовательные соединения практически не встречаются.

Эта формула актуальна для расчета общей ёмкости цепи последовательно соединенных двух конденсаторов. Для определения общей ёмкости цепи с большим числом приборов необходимо воспользоваться формулой:

Смешанная схема

Пример смешанной схемы подключения представлен ниже.

Чтобы определить общую ёмкость нескольких устройств, всю схему необходимо разделить на имеющиеся группы последовательного и параллельного соединения и рассчитать параметры ёмкости для каждой из них.

На практике данный способ встречаются на различных платах, с которыми приходиться работать радиолюбителям.

Конденсаторы, как и резисторы, можно соединять последовательно и параллельно. Рассмотрим соединение конденсаторов: для чего применяются каждая из схем, и их итоговые характеристики.

Эта схема – самая распространенная. В ней обкладки конденсаторов соединяются между собой, образуя эквивалентную емкость, равную сумме соединяемых емкостей.

При параллельном соединении электролитических конденсаторов необходимо, чтобы между собой соединялись выводы одной полярности.

Особенность такого соединения – одинаковое напряжение на всех соединяемых конденсаторах . Номинальное напряжение группы параллельно соединенных конденсаторов равно рабочему напряжению конденсатора группы, у которого оно минимально.

Токи через конденсаторы группы протекают разные: через конденсатор с большей емкостью потечет больший ток.

На практике параллельное соединение применяется для получения емкости нужной величины, когда она выходит за границы диапазона, выпускаемого промышленностью, или не укладываются в стандартный ряд емкостей. В системах регулирования коэффициента мощности (cos ϕ) изменение емкости происходит за счет автоматического подключения или отключения конденсаторов в параллель.

При последовательном соединении обкладки конденсатором соединяются друг к другу, образуя цепочку. Крайние обкладки подключаются к источнику, а ток по всем конденсаторам группы потечет одинаковый.

Эквивалентная емкость последовательно соединенных конденсаторов ограничена самой маленькой емкостью в группе. Объясняется это тем, что как только она полностью зарядится, ток прекратится. Подсчитать общую емкость двух последовательно соединенных конденсаторов можно по формуле

Но применение последовательного соединения для получения нестандартных номиналов емкостей не так распространено, как параллельного.

При последовательном соединении напряжение источника питания распределяется между конденсаторами группы. Это позволяет получить батарею конденсаторов, рассчитанную на большее напряжение , чем номинальное напряжение входящих в нее компонентов. Так из дешевых и небольших по размерам конденсаторов изготавливаются блоки, выдерживающие высокие напряжения.

Еще одна область применения последовательного соединения конденсаторов связана с перераспределением напряжений между ними. Если емкости одинаковы, напряжение делится пополам, если нет – на конденсаторе большей емкости напряжение получается большим. Устройство, работающее на этом принципе, называют емкостным делителем напряжения .

Смешанное соединение конденсаторов


Такие схемы существуют, но в устройствах специального назначения, требующие высокой точности получения величины емкости, а также для их точной настройки.

В этой статье мы попытаемся раскрыть тему соединения конденсаторов разными способам. Из статьи про соединения резисторов мы знаем,что существует последовательное, параллельное и смешанное соединение, это же правило справедливо и для этой статьи. Конденсатор (от лат. слова «condensare» — «уплотнять», «сгущать»)– это очень широко распространённый электрический прибор.

Это два проводника (обкладки), между которыми находится изоляционный материал. Если на него подать напряжение (U), то на его проводниках накопится электрический заряд(Q). Основная его характеристика – ёмкость (C). Свойства конденсатора описываются уравнением Q = UC , заряд на обкладках и напряжение прямо пропорциональны друг другу.

Условное обозначение конденсатора на схеме

Пусть на конденсатор подается переменное напряжение. Он заряжается по мере роста напряжения, электрический заряд на обкладках увеличивается. Если напряжение уменьшается, то уменьшается и заряд на его обкладках и он разряжается.

Отсюда следует, что по проводам, соединяющим конденсатор с остальной цепью, электрический ток протекает тогда, когда напряжение на конденсаторе изменяется. При этом не важно, что происходит в диэлектрике между проводниками. Сила тока равна общему заряду, протекшему в единицу времени по подключенному к конденсатору проводу. Она зависит от его емкости и скорости изменения питающего напряжения.

Ёмкость зависит от характеристик изоляции, а также размеров и формы проводника. Единица измерения ёмкости кондёра — фарада (Ф), 1 Ф=1 Кл/В. Однако на практике емкость измеряется чаще в микро- (10-6) или пико- (10-12) фарадах.

В основном используются конденсаторы для построения цепей с частотной зависимостью, для получения мощного короткого электрического импульса, там, где необходимо накапливать энергию. За счёт изменения свойств пространства между обкладками можно использовать их для измерения уровня жидкости.

Параллельное соединение

Параллельное соединение – это соединение, при котором выводы всех конденсаторов имеют две общие точки – назовём их входом и выходом схемы. Так все входы объединены в одной точке, а все выходы – в другой, напряжения на всех конденсаторах равны:

Параллельное соединение предполагает распределение полученного от источника заряда на обкладках нескольких конденсаторов, что можно записать так:

Так как напряжение на всех конденсаторах одинаковое, заряды на их обкладках зависят только от ёмкости:

Суммарная емкость параллельной группы конденсаторов:

Суммарная ёмкость такой группы конденсаторов равна сумме емкостей включенных в схему.

Блоки конденсаторов широко используются для повышения мощности и устойчивости работы энергосистем в линиях электропередач. При этом затраты на более мощные элементы линий можно снизить. Повышается стабильность работы ЛЭП, устойчивость ЛЭП к сбоям и перегрузкам.

Последовательное соединение

Последовательное соединение конденсаторов – это их подключение непосредственно друг за другом без разветвлений проводника. От источника напряжения заряды поступают на обкладки первого и последнего в цепи конденсаторов.

В силу электростатической индукции на внутренних обкладках смежных конденсаторов происходит выравнивание заряда на электрически соединённых обкладках смежных конденсаторов, поэтому на них появляются равные по величине и обратные по знаку электрические заряды.

При таком соединении электрические заряды на обкладках отдельных кондёров по величине равны:

Общее напряжение для всей цепи:

Очевидно, что напряжение между проводниками для каждого конденсатора зависит от накопленного заряда и ёмкости, т.е.:

Поэтому эквивалентная ёмкость последовательной цепи равна:

Отсюда следует, что величина, обратная общей емкости, равна сумме величин, обратных емкостям отдельных конденсаторов:

Смешанное соединение

Смешанным соединение конденсаторов называют такое соединение, при котором присутствует соединение последовательное и параллельное одновременно. Чтобы более подробно разобраться, давайте рассмотрим это соединение на примере:

На рисунке видно,что соединены два конденсатора последовательно вверху и внизу и два параллельно. Можно вывести формулу из выше описанных соединении:

Основой любой радиотехники является конденсатор, он используется в самых разнообразных схемах-это и источники питания и применение для аналоговых сигналов хранения данных, а также в телекоммуникационных связи для регулирования частоты.

В электрических цепях и схемах используются различные методы соединения конденсаторов. Соединение емкостей в конденсаторные батареи может быть последовательным, параллельным и последовательно-параллельным (смешанным).

Если подключение емкостей в батарею осуществляется в виде цепочки и к точкам включения в цепь присоединены пластины только первого и последнего конденсаторов, то такое соединение называется последовательным .

При последовательном соединение конденсаторов они заряжаются одинаковым количеством электричества, хотя от источника тока заряжаются только две крайние пластины, а остальные пластины заряжаются через влияние электрического поля. При этом заряд пластины 2 будет равен по номиналу, но противоположен по знаку заряду пластины 1, заряд пластины 3 будет равен заряду пластины 2, но также будет противоположной полярности и т. д.

Но если говорить точнее, напряжения на различных емкостных элементах будут отличаться, так как для заряда одним и тем же количеством электричества при различной номинальной емкости всегда необходимы различные напряжения. Чем нижее емкость конденсатора, тем больший уровень напряжение требуется для того, чтобы зарядить радиокомпонент необходимым количеством электричества, и наоборот.

Таким образом, при заряде группы емкостей, соединенных последовательно, на конденсаторах малой емкости напряжения будут выше, а на элементах большой емкости — ниже.

Рассмотрим всю группу емкостей соединенных последовательно, как одну эквивалентную емкость, между пластинами которой существует какой-то уровень напряжения, равный сумме напряжений на всех элементах группы, а заряд которого равен заряду любого компонента из данной группы.

Если более пристально рассмотреть самый меньший номинал емкости в группе, то на нем должно быть самый высокий уровень напряжения. Но фактически, уровень напряжения на нем составляет только часть общего значения напряжения, от общей группы. Напряжение на всей группе всегда выше напряжения на конденсаторе, имеющем самую малую велечину емкости. А поэтому можно сказать, что общая емкость группы конденсаторов, соединенных последовательно, меньше емкости самого малого конденсатора в группе .

Для вычисления общей емкости группы, в данном примере воспользуемся следующей формулой:

1 / C общ = 1/C 1 + 1/C 2 + 1/C 3

Для частного случая при двух последовательно соединенных элементов формула примет вид:

C общ = С 1 × С 2 /C 1 + C 2

Для практического примера подключим три радио компонента номиналом 100 мкф на 100 вольт последовательно. Согласно выше приведенной формуле, делим единицу, на емкость. Потом суммируем. Затем единицу делим на получившийся результат.

Итак — (1:100)+(1:100)+(1:100) = 0,01 + 0,01 + 0,01 = 0,03 и наконец 1: 0,03 = 33 мкф на 300вольт (все напряжения суммируем между собой 100+100+100 = 300в). В результате получаем конденсаторную батарею общей емкостью 33 мкф на 300 вольт.

В случае, если при последовательном соединении требуется получить неполярный конденсатор большой емкости, можно соединить два электролитических. При этом желательно выбирать конденсаторы одинакового номинала.

Включаем оба конденсатора последовательно, соединив их отрицательные электроды между собой. В итоге получим емкость равную половине каждого из номиналов

Если группа емкостных элементов включена в схему таким образом, что к точкам непосредственного включения присоединены пластины всех компонентов схемы, то такое соединение называется параллельным соединением конденсаторов.

При заряде группы емкостей, включенных параллельно, между пластинами всех элементов будет одно и тоже напряжение, так как все они заряжаются от одного источника питания. Общее количество электричества на всех элементах будет равно сумме количеств электричества, помещающихся на каждой емкости в отдельности, так как заряд каждой из них осуществляется независимо от заряда других компонентов данной схемы. Исходя из этого, всю систему можно рассматривать как один общий эквивалентный конденсатор. Тогда общая емкость при параллельном соединении конденсаторов равна сумме емкостей всех соединенных элементов.

Обозначим суммарную емкость соединенных в батарею элементов символом С общ , тогда можно записать формулу:

C общ = С 1 + С 2 + C 3

Рассмотрим эту формулу на живом примере. Предположим, что нам для ремонта бытовой техники срочно необходим конденсатор 100 мкф 50в, а у нас имеется только 47мкф на 50в. Если соединить их параллельно (минус к минусу и плюс к плюсу), то суммарная емкость получившейся конденсаторной батареи будет в районе 94 мкф на 50 вольт. Это вполне допустимое отклонение, так что можно без опаски устанавливать эту сборку в электронную технику.

Закрепим полученные знания по параллельному соединению конденсаторов на радиолюбительской практики: допустим для замены вздутого конденсатора на материнской плате персонального компьютера, нам нужна емкость номиналом 2000мкф, а у нас как назло ее не оказалось, а бежать на радиорынок тоже не хочется. Тут на помощь и придет нам знание закона параллельного соединения емкостей.

C общ = С 1 + С 2 = 1000мкф + 1000мкф = 2000мкф

Как видите нет ничего сложного, при параллельном соединении на каждый отдельный емкостной радио компонент действует одно и то же напряжение, а составной конденсатор заряжается в два раза большим количеством электричества.

Последовательно-параллельным соединением конденсаторов называется цепь или схема имеющая в своем составе участки, как с параллельным, так и с последовательным соединением радиокомпонентов.

При расчете общей емкости такой схемы с последовательно-параллельным типом соединения этот участок (как и в случае с ) разбивают на элементарные участки, состоящие из простых групп с последовательным или параллельным соединением емкостей. Дальше алгоритм вычислений принимает вид:

1. Вычисляют эквивалентную емкость участков с последовательным соединением емкостей.
2. Если эти участки состоят из последовательно соединенные конденсаторы, то сначала вычисляют их емкость.
3. После расчета эквивалентных емкостей перерисовывают схему. Обычно получается схема из последовательно соединенных эквивалентных конденсаторов.
4. Рассчитывают общую емкость полученной схемы.

Пример расчета емкости при смешанном соединение конденсаторов

Для получения большего спектра емкостей конденсаторы часто соединяют между собой, получают, так называемые батареи конденсаторов. Соединение при этом может быть параллельным, последовательным или комбинированным (смешанным). Рассмотрим случай с двумя конденсаторами.

Последовательное соединение конденсаторов показано на рис. 1

Здесь (рис.1) обкладка одного конденсатора, имеющая отрицательный заряд соединяется с положительной обкладкой следующего конденсатора. При последовательном соединении средние пластины конденсаторов электризуются через влияние, следовательно, их заряды по величине равны и противоположны по знаку. Заряды на этих конденсаторах одинаковы. При этом соединении разности потенциалов складываются:

При этом имеем:

Получаем, что при последовательном соединении конденсаторов емкость соединения находят как:

Обобщив формулу (3) для N конденсаторов, получаем:

где — электрическая емкость i-го конденсатора.

Последовательное соединение конденсаторов используют тогда, когда для избегания пробоя конденсатора необходимо разность потенциалов распределить между несколькими конденсаторами.

Последовательное соединение конденсаторов показано на рис. 2

При параллельном соединении разности потенциалов между обкладками конденсаторов одинаковы. Суммарный заряд системы равен сумме зарядов на каждом из конденсаторов:

Из сказанного выше получим:

Для батареи из N параллельно соединенных конденсаторов имеем:

Параллельное соединение конденсаторов используют тогда, когда необходимо увеличить емкость конденсатора.

Примеры решения задач

ПРИМЕР 1

ЗаданиеПолучите формулу для расчета емкости слоистого конденсатора.
РешениеКонденсатор, который называют слоистым, состоит из двух параллельных металлических обкладок, разделенных несколькими плоскими слоями разных диэлектриков (рис.3). Обозначим диэлектрические проницаемости слоев диэлектриков как . Будем считать, что соответствующая толщина слоя диэлектрика при этом: .

Допустим, что между слоями диэлектриков вставлены очень тонкие листы из проводника. От такой процедуры заряды на обкладках конденсатора и напряженности полей в солях диэлектриков останутся неизменными. Останутся без изменений разности потенциалов между обкладками, следовательно, не изменится емкость конденсатора. Но, наличие тонких листов проводника превратит слоистый конденсатор в последовательное соединение конденсаторов.

Применим формулы емкости плоского конденсатора:

и расчета емкости батареи последовательно соединенных конденсаторов:

получаем:

Ответ

ПРИМЕР 2

ЗаданиеКакой будет емкость соединения конденсаторов (рис.4), если батарея составлена из одинаковых конденсаторов, емкость каждого из них равна Ф.

РешениеЕмкость параллельного соединения конденсаторов обозначим как Она равна:

Формула расчета параллельного соединения конденсаторов. Последовательное соединение конденсаторов: формула

Содержание:

Схемы в электротехнике состоят из электрических элементов, в которых способы соединения конденсаторов могут быть разными. Надо понимать, как правильно подключить конденсатор. Отдельные участки цепи с подключенными конденсаторами можно заменить одним эквивалентным элементом. Он заменит ряд конденсаторов, но должно выполняться обязательное условие: когда напряжение, подводимое к обкладкам эквивалентного конденсатора, равняется напряжению на входе и выходе группы заменяющихся конденсаторов, тогда заряд емкости будет такой же, как и на группе емкостей. Для понимания вопроса, как подключить конденсатор в любой схеме, рассмотрим виды его включения.

Параллельное включение конденсаторов в цепь

Параллельное соединение конденсаторов — это когда все пластины подключаются к точкам включения цепи, образовывая батарею емкостей.

Разность потенциалов на пластинах накопителей емкости будет одинаковая, так как они все заряжаются от одного источника тока. В этом случае каждый заряжающийся конденсатор имеет собственный заряд при одинаковой величине, подводимой к ним энергии.

Параллельные конденсаторы, общий параметр количества заряда полученной батареи накопителей, рассчитывается, как сумма всех зарядов, помещающихся на каждой емкости, потому что каждый заряд емкости не зависит от заряда другой емкости, входящей в группу конденсаторов, параллельно включенных в схему.

При параллельном соединении конденсаторов емкость равняется:

Из представленной формулы можно сделать вывод, что всю группу накопителей можно рассматривать как один равноценный им конденсатор.

Конденсаторы, соединенные параллельно, имеют напряжение:

Последовательное включение конденсаторов в цепь

Когда в схеме выполнено последовательное соединение конденсаторов, оно выглядит как цепочка емкостных накопителей, где пластина первого и последнего накопителя емкости (конденсатора) подключены к источнику тока.

Последовательное соединение конденсатора:

При последовательном соединении конденсаторов все устройства этого участка берут одинаковое количество электроэнергии, потому что в процессе участвует первая и последняя пластинка накопителей, а пластины 2, 3 и другие до N проходят зарядку посредством влияния. По этой причине заряд пластины 2 накопителя емкости равняется по значению заряду 1 пластины, но имеет обратный знак. Заряд пластины накопителя 3 равняется значению заряда пластины 2, но так же с обратным знаком, все последующие накопители имеет аналогичную систему заряда.

Формула нахождения заряда на конденсаторе, схема подключения конденсатора:

Когда выполняется последовательное соединение конденсаторов, напряжение на каждом накопители емкости будет различное, так как в зарядке одинаковым количеством электрической энергии участвуют разные емкости. Зависимость емкости от напряжения такова: чем она меньше, тем большее напряжение необходимо подать на пластины накопителя для его зарядки. И обратная величина: чем выше емкость накопителя, тем меньше требуется напряжения для его зарядки. Можно сделать вывод, что емкость последовательно соединенных накопителей имеет значение для величины напряжения на пластинах — чем она меньше, тем больше напряжения требуется, а также накопители большой емкости требуют меньшего напряжения.

Основное отличие схемы последовательного соединения накопителей емкости в том, что электроэнергия протекает только в одном направлении, а это означает, что в каждом накопителе емкости составленной батареи ток будет одинаковым. В этом виде соединений конденсаторов обеспечивается равномерное накопление энергии независимо от емкости накопителей.

Группу накопителей емкости можно также на схеме рассматривать как эквивалентный накопитель, на пластины которого подается напряжение, определяемое формулой:

Заряд общего (эквивалентного) накопителя группы емкостных накопителей последовательного соединения равен:

Общему значению емкости последовательно соединенных конденсаторов соответствует выражение:

Смешанное включение емкостных накопителей в схему

Параллельное и последовательное соединение конденсаторов на одном из участков цепи схемы называется специалистами смешанным соединением.

Участок цепи подсоединенных смешанным включением накопителей емкости:

Смешанное соединение конденсаторов в схеме рассчитывается в определенном порядке, который можно представить следующим образом:

  • разбивается схема на простые для вычисления участки, это последовательное и параллельное соединение конденсаторов;
  • вычисляем эквивалентную емкость для группы конденсаторов, последовательно включенных на участке параллельного соединения;
  • проводим нахождение эквивалентной емкости на параллельном участке;
  • когда эквивалентные емкости накопителей определены, схему рекомендуется перерисовать;
  • рассчитывается емкость получившейся после последовательного включения эквивалентных накопителей электрической энергии.

Накопители емкостей (двухполюсники) включены разными способами в цепь, это дает несколько преимуществ в решении электротехнических задач по сравнению с традиционными способами включения конденсаторов:

  1. Использование для подключения электрических двигателей и другого оборудования в цехах, в радиотехнических устройствах.
  2. Упрощение вычисления величин электросхемы. Монтаж выполняется отдельными участками.
  3. Технические свойства всех элементов не меняются, когда изменяется сила тока и магнитное поле, это применяется для включения разных накопителей. Характеризуется постоянной величиной емкости и напряжения, а заряд пропорционален потенциалу.

Вывод

Разного вида включения конденсаторов в цепь применяются для решения электротехнических задач, в частности, для получения полярных накопителей из нескольких неполярных двухполюсников. В этом случае решением будет соединение группы однополюсных накопителей емкости по встречно-параллельному способу (треугольником). В этой схеме минус соединяется с минусом, а плюс — с плюсом. Происходит увеличение емкости накопителя, и меняется работа двухполюсника.

Не отображаются имеющиеся вхождения: последовательное параллельное и смешанное соединение конденсаторов, последовательное и параллельное соединение конденсаторов, при параллельном соединении конденсаторов емкость.

Для получения большего спектра емкостей конденсаторы часто соединяют между собой, получают, так называемые батареи конденсаторов. Соединение при этом может быть параллельным, последовательным или комбинированным (смешанным). Рассмотрим случай с двумя конденсаторами.

Последовательное соединение конденсаторов показано на рис. 1

Здесь (рис.1) обкладка одного конденсатора, имеющая отрицательный заряд соединяется с положительной обкладкой следующего конденсатора. При последовательном соединении средние пластины конденсаторов электризуются через влияние, следовательно, их заряды по величине равны и противоположны по знаку. Заряды на этих конденсаторах одинаковы. При этом соединении разности потенциалов складываются:

При этом имеем:

Получаем, что при последовательном соединении конденсаторов емкость соединения находят как:

Обобщив формулу (3) для N конденсаторов, получаем:

где — электрическая емкость i-го конденсатора.

Последовательное соединение конденсаторов используют тогда, когда для избегания пробоя конденсатора необходимо разность потенциалов распределить между несколькими конденсаторами.

Последовательное соединение конденсаторов показано на рис. 2

При параллельном соединении разности потенциалов между обкладками конденсаторов одинаковы. Суммарный заряд системы равен сумме зарядов на каждом из конденсаторов:

Из сказанного выше получим:

Для батареи из N параллельно соединенных конденсаторов имеем:

Параллельное соединение конденсаторов используют тогда, когда необходимо увеличить емкость конденсатора.

Примеры решения задач

ПРИМЕР 1

ЗаданиеПолучите формулу для расчета емкости слоистого конденсатора.
РешениеКонденсатор, который называют слоистым, состоит из двух параллельных металлических обкладок, разделенных несколькими плоскими слоями разных диэлектриков (рис.3). Обозначим диэлектрические проницаемости слоев диэлектриков как . Будем считать, что соответствующая толщина слоя диэлектрика при этом: .

Допустим, что между слоями диэлектриков вставлены очень тонкие листы из проводника. От такой процедуры заряды на обкладках конденсатора и напряженности полей в солях диэлектриков останутся неизменными. Останутся без изменений разности потенциалов между обкладками, следовательно, не изменится емкость конденсатора. Но, наличие тонких листов проводника превратит слоистый конденсатор в последовательное соединение конденсаторов.

Применим формулы емкости плоского конденсатора:

и расчета емкости батареи последовательно соединенных конденсаторов:

получаем:

Ответ

ПРИМЕР 2

ЗаданиеКакой будет емкость соединения конденсаторов (рис.4), если батарея составлена из одинаковых конденсаторов, емкость каждого из них равна Ф.

РешениеЕмкость параллельного соединения конденсаторов обозначим как Она равна:

В этой статье мы попытаемся раскрыть тему соединения конденсаторов разными способам. Из статьи про соединения резисторов мы знаем,что существует последовательное, параллельное и смешанное соединение, это же правило справедливо и для этой статьи. Конденсатор (от лат. слова «condensare» — «уплотнять», «сгущать»)– это очень широко распространённый электрический прибор.

Это два проводника (обкладки), между которыми находится изоляционный материал. Если на него подать напряжение (U), то на его проводниках накопится электрический заряд(Q). Основная его характеристика – ёмкость (C). Свойства конденсатора описываются уравнением Q = UC , заряд на обкладках и напряжение прямо пропорциональны друг другу.

Условное обозначение конденсатора на схеме

Пусть на конденсатор подается переменное напряжение. Он заряжается по мере роста напряжения, электрический заряд на обкладках увеличивается. Если напряжение уменьшается, то уменьшается и заряд на его обкладках и он разряжается.

Отсюда следует, что по проводам, соединяющим конденсатор с остальной цепью, электрический ток протекает тогда, когда напряжение на конденсаторе изменяется. При этом не важно, что происходит в диэлектрике между проводниками. Сила тока равна общему заряду, протекшему в единицу времени по подключенному к конденсатору проводу. Она зависит от его емкости и скорости изменения питающего напряжения.

Ёмкость зависит от характеристик изоляции, а также размеров и формы проводника. Единица измерения ёмкости кондёра — фарада (Ф), 1 Ф=1 Кл/В. Однако на практике емкость измеряется чаще в микро- (10-6) или пико- (10-12) фарадах.

В основном используются конденсаторы для построения цепей с частотной зависимостью, для получения мощного короткого электрического импульса, там, где необходимо накапливать энергию. За счёт изменения свойств пространства между обкладками можно использовать их для измерения уровня жидкости.

Параллельное соединение

Параллельное соединение – это соединение, при котором выводы всех конденсаторов имеют две общие точки – назовём их входом и выходом схемы. Так все входы объединены в одной точке, а все выходы – в другой, напряжения на всех конденсаторах равны:

Параллельное соединение предполагает распределение полученного от источника заряда на обкладках нескольких конденсаторов, что можно записать так:

Так как напряжение на всех конденсаторах одинаковое, заряды на их обкладках зависят только от ёмкости:

Суммарная емкость параллельной группы конденсаторов:

Суммарная ёмкость такой группы конденсаторов равна сумме емкостей включенных в схему.

Блоки конденсаторов широко используются для повышения мощности и устойчивости работы энергосистем в линиях электропередач. При этом затраты на более мощные элементы линий можно снизить. Повышается стабильность работы ЛЭП, устойчивость ЛЭП к сбоям и перегрузкам.

Последовательное соединение

Последовательное соединение конденсаторов – это их подключение непосредственно друг за другом без разветвлений проводника. От источника напряжения заряды поступают на обкладки первого и последнего в цепи конденсаторов.

В силу электростатической индукции на внутренних обкладках смежных конденсаторов происходит выравнивание заряда на электрически соединённых обкладках смежных конденсаторов, поэтому на них появляются равные по величине и обратные по знаку электрические заряды.

При таком соединении электрические заряды на обкладках отдельных кондёров по величине равны:

Общее напряжение для всей цепи:

Очевидно, что напряжение между проводниками для каждого конденсатора зависит от накопленного заряда и ёмкости, т.е.:

Поэтому эквивалентная ёмкость последовательной цепи равна:

Отсюда следует, что величина, обратная общей емкости, равна сумме величин, обратных емкостям отдельных конденсаторов:

Смешанное соединение

Смешанным соединение конденсаторов называют такое соединение, при котором присутствует соединение последовательное и параллельное одновременно. Чтобы более подробно разобраться, давайте рассмотрим это соединение на примере:

На рисунке видно,что соединены два конденсатора последовательно вверху и внизу и два параллельно. Можно вывести формулу из выше описанных соединении:

Основой любой радиотехники является конденсатор, он используется в самых разнообразных схемах-это и источники питания и применение для аналоговых сигналов хранения данных, а также в телекоммуникационных связи для регулирования частоты.

Конденсаторы, как и резисторы, можно соединять последовательно и параллельно. Рассмотрим соединение конденсаторов: для чего применяются каждая из схем, и их итоговые характеристики.

Эта схема – самая распространенная. В ней обкладки конденсаторов соединяются между собой, образуя эквивалентную емкость, равную сумме соединяемых емкостей.

При параллельном соединении электролитических конденсаторов необходимо, чтобы между собой соединялись выводы одной полярности.

Особенность такого соединения – одинаковое напряжение на всех соединяемых конденсаторах . Номинальное напряжение группы параллельно соединенных конденсаторов равно рабочему напряжению конденсатора группы, у которого оно минимально.

Токи через конденсаторы группы протекают разные: через конденсатор с большей емкостью потечет больший ток.

На практике параллельное соединение применяется для получения емкости нужной величины, когда она выходит за границы диапазона, выпускаемого промышленностью, или не укладываются в стандартный ряд емкостей. В системах регулирования коэффициента мощности (cos ϕ) изменение емкости происходит за счет автоматического подключения или отключения конденсаторов в параллель.

При последовательном соединении обкладки конденсатором соединяются друг к другу, образуя цепочку. Крайние обкладки подключаются к источнику, а ток по всем конденсаторам группы потечет одинаковый.

Эквивалентная емкость последовательно соединенных конденсаторов ограничена самой маленькой емкостью в группе. Объясняется это тем, что как только она полностью зарядится, ток прекратится. Подсчитать общую емкость двух последовательно соединенных конденсаторов можно по формуле

Но применение последовательного соединения для получения нестандартных номиналов емкостей не так распространено, как параллельного.

При последовательном соединении напряжение источника питания распределяется между конденсаторами группы. Это позволяет получить батарею конденсаторов, рассчитанную на большее напряжение , чем номинальное напряжение входящих в нее компонентов. Так из дешевых и небольших по размерам конденсаторов изготавливаются блоки, выдерживающие высокие напряжения.

Еще одна область применения последовательного соединения конденсаторов связана с перераспределением напряжений между ними. Если емкости одинаковы, напряжение делится пополам, если нет – на конденсаторе большей емкости напряжение получается большим. Устройство, работающее на этом принципе, называют емкостным делителем напряжения .

Смешанное соединение конденсаторов


Такие схемы существуют, но в устройствах специального назначения, требующие высокой точности получения величины емкости, а также для их точной настройки.

В электротехнике существуют различные варианты подключения электрических элементов. В частности, существует последовательное, параллельное или смешанное соединение конденсаторов, в зависимости от потребностей схемы. Рассмотрим их.

Параллельное соединение

Параллельное соединение характеризуется тем, что все пластины электрических конденсаторов присоединяются к точкам включения и образовывают собой батареи. В таком случае, во время заряда конденсаторов каждый из них будет иметь различное число электрических зарядов при одинаковом количестве подводимой энергии

Схема параллельного крепления

Емкость при параллельной установке рассчитывается исходя из емкостей всех конденсаторов в схеме. При этом, количество электрической энергии, поступающей на все отдельные двухполюсные элементы цепи, можно будет рассчитать, суммировав сумму энергии, помещающейся в каждый конденсатор. Вся схема, подключенная таким образом, рассчитывается как один двухполюсник.

C общ = C 1 + C 2 + C 3


Схема – напряжение на накопителях

В отличие от соединения звездой, на обкладки всех конденсаторов попадает одинаковое напряжение. Например, на схеме выше мы видим, что:

V AB = V C1 = V C2 = V C3 = 20 Вольт

Последовательное соединение

Здесь к точкам включения присоединяются контакты только первого и последнего конденсатора.


Схема – схема последовательного соединения

Главной особенностью работы схемы является то, что электрическая энергия будет проходить только по одному направлению, значит, что в каждом из конденсаторов ток будет одинаковым. В такой цепи для каждого накопителя, независимо от его емкости, будет обеспечиваться равное накопление проходящей энергии. Нужно понимать, что каждый из них последовательно соприкасается со следующим и предыдущим, а значит, емкость при последовательном типе может воспроизводиться энергией соседнего накопителя.

Формула, которая отражает зависимость тока от соединения конденсаторов, имеет такой вид:

i = i c 1 = i c 2 = i c 3 = i c 4 , то есть токи проходящие через каждый конденсатор равны между собой.

Следовательно, одинаковой будет не только сила тока, но и электрический заряд. По формуле это определяется как:

Q общ = Q 1 = Q 2 = Q 3

А так определяется общая суммарная емкость конденсаторов при последовательном соединении:

1/C общ = 1/C 1 + 1/C 2 + 1/C 3

Видео: как соединять конденсаторы параллельным и последовательным методом

Смешанное подключение

Но, стоит учитывать, что для соединения различных конденсаторов необходимо учитывать напряжение сети. Для каждого полупроводника этот показатель будет отличаться в зависимости от емкости элемента. Отсюда следует, что отдельные группы полупроводниковых двухполюсников малой емкости будут при зарядке становиться больше, и наоборот, электроемкость большого размера будет нуждаться в меньшем заряде.


Схема: смешанное соединение конденсаторов

Существует также смешанное соединение двух и более конденсаторов. Здесь электрическая энергия распределяется одновременно при помощи параллельного и последовательного подключения электролитических элементов в цепь. Эта схема имеет несколько участков с различным подключением конденсирующих двухполюсников. Иными словами, на одном цепь параллельно включена, на другом – последовательно. Такая электрическая схема имеет ряд достоинств сравнительно с традиционными:

  1. Можно использовать для любых целей: подключения электродвигателя, станочного оборудования, радиотехнических приборов;
  2. Простой расчет. Для монтажа вся схема разбивается на отдельные участки цепи, которые рассчитываются по отдельности;
  3. Свойства компонентов не изменяются независимо от изменений электромагнитного поля, силы тока. Это очень важно при работе с разноименными двухполюсниками. Ёмкость постоянна при постоянном напряжении, но, при этом, потенциал пропорционален заряду;
  4. Если требуется собрать несколько неполярных полупроводниковых двухполюсников из полярных, то нужно взять несколько однополюсных двухполюсника и соединить их встречно-параллельным способом (в треугольник). Минус к минусу, а плюс к плюсу. Таким образом, за счет увеличения емкости изменяется принцип работы двухполюсного полупроводника.

Соединение конденсаторов формула. Последовательное соединение конденсаторов: формула

В электротехнике существуют различные варианты подключения электрических элементов. В частности, существует последовательное, параллельное или смешанное соединение конденсаторов, в зависимости от потребностей схемы. Рассмотрим их.

Параллельное соединение

Параллельное соединение характеризуется тем, что все пластины электрических конденсаторов присоединяются к точкам включения и образовывают собой батареи. В таком случае, во время заряда конденсаторов каждый из них будет иметь различное число электрических зарядов при одинаковом количестве подводимой энергии

Схема параллельного крепления

Емкость при параллельной установке рассчитывается исходя из емкостей всех конденсаторов в схеме. При этом, количество электрической энергии, поступающей на все отдельные двухполюсные элементы цепи, можно будет рассчитать, суммировав сумму энергии, помещающейся в каждый конденсатор. Вся схема, подключенная таким образом, рассчитывается как один двухполюсник.

C общ = C 1 + C 2 + C 3


Схема – напряжение на накопителях

В отличие от соединения звездой, на обкладки всех конденсаторов попадает одинаковое напряжение. Например, на схеме выше мы видим, что:

V AB = V C1 = V C2 = V C3 = 20 Вольт

Последовательное соединение

Здесь к точкам включения присоединяются контакты только первого и последнего конденсатора.


Схема – схема последовательного соединения

Главной особенностью работы схемы является то, что электрическая энергия будет проходить только по одному направлению, значит, что в каждом из конденсаторов ток будет одинаковым. В такой цепи для каждого накопителя, независимо от его емкости, будет обеспечиваться равное накопление проходящей энергии. Нужно понимать, что каждый из них последовательно соприкасается со следующим и предыдущим, а значит, емкость при последовательном типе может воспроизводиться энергией соседнего накопителя.

Формула, которая отражает зависимость тока от соединения конденсаторов, имеет такой вид:

i = i c 1 = i c 2 = i c 3 = i c 4 , то есть токи проходящие через каждый конденсатор равны между собой.

Следовательно, одинаковой будет не только сила тока, но и электрический заряд. По формуле это определяется как:

Q общ = Q 1 = Q 2 = Q 3

А так определяется общая суммарная емкость конденсаторов при последовательном соединении:

1/C общ = 1/C 1 + 1/C 2 + 1/C 3

Видео: как соединять конденсаторы параллельным и последовательным методом

Смешанное подключение

Но, стоит учитывать, что для соединения различных конденсаторов необходимо учитывать напряжение сети. Для каждого полупроводника этот показатель будет отличаться в зависимости от емкости элемента. Отсюда следует, что отдельные группы полупроводниковых двухполюсников малой емкости будут при зарядке становиться больше, и наоборот, электроемкость большого размера будет нуждаться в меньшем заряде.


Схема: смешанное соединение конденсаторов

Существует также смешанное соединение двух и более конденсаторов. Здесь электрическая энергия распределяется одновременно при помощи параллельного и последовательного подключения электролитических элементов в цепь. Эта схема имеет несколько участков с различным подключением конденсирующих двухполюсников. Иными словами, на одном цепь параллельно включена, на другом – последовательно. Такая электрическая схема имеет ряд достоинств сравнительно с традиционными:

  1. Можно использовать для любых целей: подключения электродвигателя, станочного оборудования, радиотехнических приборов;
  2. Простой расчет. Для монтажа вся схема разбивается на отдельные участки цепи, которые рассчитываются по отдельности;
  3. Свойства компонентов не изменяются независимо от изменений электромагнитного поля, силы тока. Это очень важно при работе с разноименными двухполюсниками. Ёмкость постоянна при постоянном напряжении, но, при этом, потенциал пропорционален заряду;
  4. Если требуется собрать несколько неполярных полупроводниковых двухполюсников из полярных, то нужно взять несколько однополюсных двухполюсника и соединить их встречно-параллельным способом (в треугольник). Минус к минусу, а плюс к плюсу. Таким образом, за счет увеличения емкости изменяется принцип работы двухполюсного полупроводника.

Содержание:

Схемы в электротехнике состоят из электрических элементов, в которых способы соединения конденсаторов могут быть разными. Надо понимать, как правильно подключить конденсатор. Отдельные участки цепи с подключенными конденсаторами можно заменить одним эквивалентным элементом. Он заменит ряд конденсаторов, но должно выполняться обязательное условие: когда напряжение, подводимое к обкладкам эквивалентного конденсатора, равняется напряжению на входе и выходе группы заменяющихся конденсаторов, тогда заряд емкости будет такой же, как и на группе емкостей. Для понимания вопроса, как подключить конденсатор в любой схеме, рассмотрим виды его включения.

Параллельное включение конденсаторов в цепь

Параллельное соединение конденсаторов — это когда все пластины подключаются к точкам включения цепи, образовывая батарею емкостей.

Разность потенциалов на пластинах накопителей емкости будет одинаковая, так как они все заряжаются от одного источника тока. В этом случае каждый заряжающийся конденсатор имеет собственный заряд при одинаковой величине, подводимой к ним энергии.

Параллельные конденсаторы, общий параметр количества заряда полученной батареи накопителей, рассчитывается, как сумма всех зарядов, помещающихся на каждой емкости, потому что каждый заряд емкости не зависит от заряда другой емкости, входящей в группу конденсаторов, параллельно включенных в схему.

При параллельном соединении конденсаторов емкость равняется:

Из представленной формулы можно сделать вывод, что всю группу накопителей можно рассматривать как один равноценный им конденсатор.

Конденсаторы, соединенные параллельно, имеют напряжение:

Последовательное включение конденсаторов в цепь

Когда в схеме выполнено последовательное соединение конденсаторов, оно выглядит как цепочка емкостных накопителей, где пластина первого и последнего накопителя емкости (конденсатора) подключены к источнику тока.

Последовательное соединение конденсатора:

При последовательном соединении конденсаторов все устройства этого участка берут одинаковое количество электроэнергии, потому что в процессе участвует первая и последняя пластинка накопителей, а пластины 2, 3 и другие до N проходят зарядку посредством влияния. По этой причине заряд пластины 2 накопителя емкости равняется по значению заряду 1 пластины, но имеет обратный знак. Заряд пластины накопителя 3 равняется значению заряда пластины 2, но так же с обратным знаком, все последующие накопители имеет аналогичную систему заряда.

Формула нахождения заряда на конденсаторе, схема подключения конденсатора:

Когда выполняется последовательное соединение конденсаторов, напряжение на каждом накопители емкости будет различное, так как в зарядке одинаковым количеством электрической энергии участвуют разные емкости. Зависимость емкости от напряжения такова: чем она меньше, тем большее напряжение необходимо подать на пластины накопителя для его зарядки. И обратная величина: чем выше емкость накопителя, тем меньше требуется напряжения для его зарядки. Можно сделать вывод, что емкость последовательно соединенных накопителей имеет значение для величины напряжения на пластинах — чем она меньше, тем больше напряжения требуется, а также накопители большой емкости требуют меньшего напряжения.

Основное отличие схемы последовательного соединения накопителей емкости в том, что электроэнергия протекает только в одном направлении, а это означает, что в каждом накопителе емкости составленной батареи ток будет одинаковым. В этом виде соединений конденсаторов обеспечивается равномерное накопление энергии независимо от емкости накопителей.

Группу накопителей емкости можно также на схеме рассматривать как эквивалентный накопитель, на пластины которого подается напряжение, определяемое формулой:

Заряд общего (эквивалентного) накопителя группы емкостных накопителей последовательного соединения равен:

Общему значению емкости последовательно соединенных конденсаторов соответствует выражение:

Смешанное включение емкостных накопителей в схему

Параллельное и последовательное соединение конденсаторов на одном из участков цепи схемы называется специалистами смешанным соединением.

Участок цепи подсоединенных смешанным включением накопителей емкости:

Смешанное соединение конденсаторов в схеме рассчитывается в определенном порядке, который можно представить следующим образом:

  • разбивается схема на простые для вычисления участки, это последовательное и параллельное соединение конденсаторов;
  • вычисляем эквивалентную емкость для группы конденсаторов, последовательно включенных на участке параллельного соединения;
  • проводим нахождение эквивалентной емкости на параллельном участке;
  • когда эквивалентные емкости накопителей определены, схему рекомендуется перерисовать;
  • рассчитывается емкость получившейся после последовательного включения эквивалентных накопителей электрической энергии.

Накопители емкостей (двухполюсники) включены разными способами в цепь, это дает несколько преимуществ в решении электротехнических задач по сравнению с традиционными способами включения конденсаторов:

  1. Использование для подключения электрических двигателей и другого оборудования в цехах, в радиотехнических устройствах.
  2. Упрощение вычисления величин электросхемы. Монтаж выполняется отдельными участками.
  3. Технические свойства всех элементов не меняются, когда изменяется сила тока и магнитное поле, это применяется для включения разных накопителей. Характеризуется постоянной величиной емкости и напряжения, а заряд пропорционален потенциалу.

Вывод

Разного вида включения конденсаторов в цепь применяются для решения электротехнических задач, в частности, для получения полярных накопителей из нескольких неполярных двухполюсников. В этом случае решением будет соединение группы однополюсных накопителей емкости по встречно-параллельному способу (треугольником). В этой схеме минус соединяется с минусом, а плюс — с плюсом. Происходит увеличение емкости накопителя, и меняется работа двухполюсника.

Не отображаются имеющиеся вхождения: последовательное параллельное и смешанное соединение конденсаторов, последовательное и параллельное соединение конденсаторов, при параллельном соединении конденсаторов емкость.

Рис.2 U=U 1 =U 2 =U 3

    Общий заряд Q всех конденсаторов

    Общая емкость С, или емкость батареи, параллельно включенных конденсаторов равна сумме емкостей этих конденсаторов.

Параллельное подключение конденсатора к группе других включенных конденсаторов увеличивает общую емкость батареи этих конденсаторов. Следовательно, параллельное соединение конденсаторов при­меняется для увеличения емкости.

4)Если параллельно включены т одинаковых конденсаторов ем­костью С´ каждый, то общая (эквивалентная) емкость батареи этих конденсаторов может быть определена выражением

Последовательное соединение конденсаторов

Рис.3

    На обкладках последовательно соединенных конденсаторов, подключенных к источнику постоянного тока с напряжением U , появятся заряды одинаковые по величине с противоположными знаками.

    Напряжение на конденсаторах распределяется обратно пропорционально емкостям конденса­торов:

    Обратная величина общей емкости последовательно соединенных конденсаторов равна сумме обратных величин емкостей этих кон­денсаторов.

При последовательном включении двух конденсаторов их об­щая емкость определяется следующим выражением:

Если в цепь включены последовательно п одинаковых конден­саторов емкостью С каждый, то общая емкость этих конденса­торов:

Из (14) видно, что, чем больше конденсаторов п соединено последовательно, тем меньше будет их общая емкость С, т. е. по­следовательное включение конденсаторов приводит к уменьше­нию общей емкости батареи конденсаторов.

На практике может оказаться, что допустимое ра­бочее напряжение U p конденсатора меньше напряжения, на кото­рое необходимо подключить конденсатор. Если этот конденсатор подключить на такое напряжение, то он выйдет из строя, так как будет пробит диэлектрик. Если же последовательно включить не­сколько конденсаторов, то напряжение распределится между ними и на каждом конденсаторе напряжение окажется мень­ше его допустимого рабочего U p . Следовательно, последовательное соединение конденсаторов применяют для того, чтобы напряжение на каждом конденсаторе не превышало его рабочего напряжения U p .

Смешанное соединение конденсаторов

Смешанное соединение (последовательно-параллельное) кон­денсаторов применяют тогда, когда необходимо увеличить ем­кость и рабочее напряжение батареи конденсаторов.

Рассмотрим смешанное соединение конденсаторов на ниже­приведенных примерах.

Энергия конденсаторов


где Q — заряд конденсатора или конденсаторов, к которым при­ложено напряжение U ; С — электрическая емкость конденсатора или батареи соединенных конденсаторов, к которой приложено напряжение U .

Таким образом, конденсаторы служат для накопления и сохра­нения электрического поля и его энергии.

15. Дайте определение понятиям трех лучевая звезда и треугольник сопротивлений. Запишите формулы для преобразования трех лучевой звезды сопротивлений в треугольник сопротивлений и наоборот. Преобразуйте схему к двум узлам (Рисунок 5)

Рисунок 5- Схема электрическая

6.СХЕМЫ ЗАМЕЩЕНИЯ

Для облегчения расчета составляется схема замещения электрической цепи, т. е. схема, отображающая свойства цепи при определенных условиях.

На схеме замещения изображают все элементы, влиянием которых на результат расчета нельзя пренебречь, и указывают также электрические соединения между ними, которые имеются в цепи.

1.Схемы замещения элементов электрических цепей

На расчетных схемах источник энергии можно представить ЭДС без внутреннего сопротивления, если это сопротивление мало по сравнению с сопротивлением приемника (рис. 3.13,6).

Приr= 0 внутреннее падение напряженияUо = 0, поэтому

напряжение на зажимах источника при любом токе равно

ЭДС: U = E = const.

В некоторых случаях источник электрической энергии на расчетной схеме заменяют другой (эквивалентной) схемой (рис. 3.14, а), где вместо ЭДСЕ источник характеризуется его током короткого замыканияI K , а вместо внутреннего со­противления в расчет вводится внутренняя проводимостьg =1/ r .

Возможность такой замены можно доказать, разделив равенство (3.1) на r:

U / r = E / r I ,

где U / r = Io -некоторый ток, равный отношению напряжения на зажимах источника к внутреннему сопротивлению;E / r = I K — ток короткого замыкания источника;

Вводя новые обозначения, получим равенство I K = Io + I , которому удовлетворяет эквивалентная схема рис. 3.14,а.

В этом случае при любой величине напряжения на зажимах; источника его ток остается равным току короткого замыкания (рис. 3.14,6):

Источник с неизменным током, не зависящим от внешнего сопротивления, называют источником тока.

Один и тот же источник электрической энергии может быть заменен в расчетной схеме источником ЭДС или источником тока.

Под последовательным соединением подразумевают случаи, когда два или больше элемента имеют вид цепи, при этом каждый из них соединяется с другим только в одной точке. Зачем конденсаторы так размещаются? Как это правильно сделать? Что необходимо знать? Какие особенности последовательное соединение конденсаторов имеет на практике? Какая формула результата?

Что необходимо знать для правильного соединения?

Увы, но здесь не всё так легко сделать, как может показаться. Многие новички думают, что если на схематическом рисунке написано, что необходим элемент на 49 микрофарад, то достаточно его просто взять и установить (или заменить равнозначным). Но необходимые параметры подобрать сложно даже в профессиональной мастерской. И что делать, если нет нужных элементов? Допустим, есть такая ситуация: необходим конденсатор на 100 микрофарад, а есть несколько штук на 47. Поставить его не всегда можно. Ехать на радиорынок за одним конденсатором? Не обязательно. Достаточно будет соединить пару элементов. Существует два основных способа: последовательное и параллельное соединение конденсаторов. Вот о первом мы и поговорим. Но если говорить про последовательное соединение катушки и конденсатора, то тут особых проблем нет.

Зачем так делают?

Когда с ними проводятся такие манипуляции, то электрические заряды на обкладках отдельных элементов будут равны: КЕ=К 1 =К 2 =К 3 . КЕ — конечная емкость, К — пропускаемое значение конденсатора. Почему так? Когда заряды поступают от источника питания на внешние обкладки, то на внутренних может быть осуществлен перенос величины, которая является значением элемента с наименьшими параметрами. То есть если взять конденсатор на 3 мкФ, а после него подсоединить на 1 мкФ — то конечный результат будет 1 мкФ. Конечно, на первом можно будет наблюдать значение в 3 мкФ. Но второй элемент не сможет столько пропустить, и он будет срезать всё, что больше необходимого значения, оставляя большую емкость на первоначальном конденсаторе. Давайте рассмотрим, что нужно рассчитать, когда делается последовательное соединение конденсаторов. Формула:

  • ОЕ — общая емкость;
  • Н — напряжение;
  • КЕ — конечная емкость.

Что ещё необходимо знать, чтобы правильно соединить конденсаторы?

Для начала не забывайте, что кроме ёмкости они ещё обладают номинальным напряжением. Почему? Когда осуществляется последовательное соединение, то напряжение распределяется обратно пропорционально их ёмкостям между ними самими. Поэтому использовать такой подход имеет смысл только в тех случаях, когда любой конденсатор сможет предоставить минимально необходимые параметры работы. Если используются элементы, у которых одинаковая емкость, то напряжение между ними будет разделяться поровну. Также небольшое предостережение относительно электролитических конденсаторов: при работе с ними всегда внимательно контролируйте их полярность. Ибо при игнорировании этого фактора последовательное соединение конденсаторов может дать ряд нежелательных эффектов. И хорошо, если всё ограничится только пробоем данных элементов. Помните, что конденсаторы копят ток, и если что-то пойдёт не так, в зависимости от схемы может случиться прецедент, в результате которого из строя выйдут другие составляющие схемы.

Ток при последовательном соединении

Из-за того, что у него существует только один возможный путь протекания, он будет иметь одно значение для всех конденсаторов. При этом количество накопленного заряда везде обладает одинаковым значением. От емкости это не зависит. Посмотрите на любую схему последовательного соединения конденсаторов. Правая обкладка первого соединена с левой второго и так далее. Если используется больше 1 элемента, то часть из них будет изолированной от общей цепи. Таким образом, эффективная площадь обкладок становится меньшей и равняется параметрам самого маленького конденсатора. Какое физическое явление лежит в основе этого процесса? Дело в том, что как только конденсатор наполняется электрическим зарядом, то он перестаёт пропускать ток. И он тогда не может протекать по всей цепи. Остальные конденсаторы в таком случае тоже не смогут заряжаться.

Падение напряженности и общая емкость

Каждый элемент понемногу рассеивает напряжение. Учитывая, что емкость ему обратно пропорциональна, то чем она меньше, тем большим будет падение. Как уже упоминалось ранее, последовательно соединённые конденсаторы обладают одинаковым электрическим зарядом. Поэтому при делении всех выражений на общее значение можно получить уравнение, которое покажет всю емкость. В этом последовательное и параллельное соединение конденсаторов сильно разнятся.

Пример № 1

Давайте воспользуемся представленными в статье формулами и рассчитаем несколько практических задач. Итак, у нас есть три конденсатора. Их емкость составляет: С1 = 25 мкФ, С2 = 30 мкФ и С3 = 20 мкФ. Они соединены последовательно. Необходимо найти их общую емкость. Используем соответствующее уравнение 1/С: 1/С1 + 1/С2 + 1/С3 = 1/25 + 1/30 + 1/20 = 37/300. Переводим в микрофарады, и общая емкость конденсатора при последовательном соединении (а группа в данном случае считается как один элемент) составляет примерно 8,11 мкФ.

Пример № 2

Давайте, чтобы закрепить наработки, решим ещё одну задачу. Имеется 100 конденсаторов. Емкость каждого элемента составляет 2 мкФ. Необходимо определить их общую емкость. Нужно их количество умножить на характеристику: 100*2=200 мкФ. Итак, общая емкость конденсатора при последовательном соединении составляет 200 микрофарад. Как видите, ничего сложного.

Заключение

Итак, мы проработали теоретические аспекты, разобрали формулы и особенности правильного соединения конденсаторов (последовательно) и даже решили несколько задачек. Хочется напомнить, чтобы читатели не упускали из внимания влияние номинального напряжения. Также желательно, чтобы подбирались элементы одного типа (слюдяные, керамические, металлобумажные, плёночные). Тогда последовательное соединение конденсаторов сможет дать нам наибольший полезный эффект.

У многих радиолюбителей, особенно приступающих впервые к конструированию электросхем, возникает вопрос, как надо подключить конденсатор требуемой ёмкости? Когда, к примеру, в каком-то месте схемы нужен конденсатор ёмкостью 470 мкФ, и такой элемент есть в наличии, то проблемы не возникнет. Но когда требуется поставить конденсатор на 1000 мкФ, а присутствуют только элементы неподходящей емкости, на помощь приходят схемы из нескольких конденсаторов, соединённых вместе. Соединять элементы можно, применяя параллельное и последовательное соединение конденсаторов по отдельности или по комбинированному принципу.

Jpg?.jpg 600w, https://elquanta.ru/wp-content/uploads/2018/04/1-21-768×410..jpg 260w, https://elquanta.ru/wp-content/uploads/2018/04/1-21.jpg 960w»>

Последовательное соединение конденсаторов

Схема последовательного соединения

Когда применяется схема последовательного соединения конденсаторов, заряд каждой детали эквивалентен. С источником соединены только внешние пластины, другие – заряжаются перераспределением электрозарядов между ними. Все конденсаторы сохраняют аналогичное количество заряда на своих обкладках. Это объясняется тем, что на каждый последующий элемент поступает заряд от соседнего. Вследствие этого справедливо уравнение:

q = q1 = q2 = q3 = …

Известно, что при последовательном соединении резисторных элементов их сопротивления суммируются, но емкость конденсатора, включенного в такую электроцепь, рассчитывается по-другому.

Падение напряжения на отдельном конденсаторном элементе зависит от его емкости. Если в последовательной электроцепи имеется три конденсаторных элемента, составляется выражение для напряжения U на основании закона Кирхгофа:

U = U1 + U2 + U3,

при этом U= q/C, U1 = q/C1, U2 = q/C2, U3 = q/C3.

Подставляя значения для напряжений в обе части уравнения, получается:

q/C = q/C1 + q/C2 + q/C3.

Так как электрозаряд q – величина одинаковая, на нее можно поделить все части полученного выражения.

Результирующая формула для емкостей конденсаторов:

1/С = 1/С1 + 1/С2 + 1/С3.

Важно! Если конденсаторы подключаются в последовательную электроцепь, показатель, обратный результирующей емкости, равен совокупности обратных значений единичных емкостей.

Jpg?.jpg 600w, https://elquanta.ru/wp-content/uploads/2018/04/2-20-768×476..jpg 120w, https://elquanta.ru/wp-content/uploads/2018/04/2-20.jpg 913w»>

Особенности последовательного соединения

Пример. Три конденсаторных элемента подключены в последовательную цепь и обладают емкостями: С1 = 0,05 мкф, С2 = 0,2 мкФ, С3 = 0,4 мкФ. Рассчитать общую емкостную величину:

  1. 1/С = 1/0,05 + 1/0,2 + 1/0,4 = 27,5;
  2. С = 1/27,5 = 0,036 мкФ.

Важно! Когда конденсаторные элементы включены в последовательную электроцепь, общее емкостное значение не превышает наименьшей емкости отдельного элемента.

Если цепь состоит всего из двух компонентов, формула переписывается в таком виде:

С = (С1 х С2)/(С1 + С2).

В случае создания цепи из двух конденсаторов с идентичным емкостным значением:

С = (С х С)/(2 х С) = С/2.

Последовательно включенные конденсаторы имеют реактивное сопротивление, зависящее от частоты протекающего тока. На каждом конденсаторе напряжение падает из-за наличия этого сопротивления, поэтому на основе такой схемы создается емкостной делитель напряжения.

Png?x15027″ alt=»Емкостной делитель напряжения»>

Емкостной делитель напряжения

Формула для емкостного делителя напряжения:

U1 = U x C/C1, U2 = U x C/C2, где:

  • U – напряжение питания схемы;
  • U1, U2 – падение напряжения на каждом элементе;
  • С – итоговая емкость схемы;
  • С1, С2 – емкостные показатели единичных элементов.

Вычисление падений напряжения на конденсаторах

К примеру, имеются сеть переменного тока 12 В и две альтернативных электроцепи подсоединения последовательных конденсаторных элементов:

  • первая – для подключения одного конденсатора С1 = 0,1 мкФ, другого С2 = 0,5 мкФ;
  • вторая – С1 = С2 = 400 нФ.
Первый вариант
  1. Итоговая емкость электросхемы С = (С1 х С2)/(С1 + С2) = 0,1 х 0,5/(0,1 + 0,5) = 0,083 мкФ;
  2. Падение напряжения на одном конденсаторе: U1 = U x C/C1 = 12 x 0,083/0,1 = 9,9 В
  3. На втором конденсаторе: U2 = U x C/C2 = 12 х 0,083/0,5 = 1,992 В.
Второй вариант
  1. Результирующая емкость С = 400 х 400/(400 + 400) = 200 нФ;
  2. Падение напряжения U1 = U2 = 12 x 200/400 = 6 В.

Согласно расчетам, можно сделать выводы, что если подключаются конденсаторы равных емкостей, вольтаж делится поровну на обоих элементах, а когда емкостные значения различаются, то на конденсаторе с меньшей емкостной величиной напряжение увеличивается, и наоборот.

Параллельное и комбинированное соединение

Параллельное соединение конденсаторов представляется иным уравнением. Для определения общего емкостного значения надо просто найти совокупность всех величин по отдельности:

С = С1 + С2 + С3 + …

Напряжение к каждому элементу будет прикладываться идентичное. Следовательно, для усиления емкости надо соединить несколько деталей параллельно.

Если соединения смешанные, последовательно-параллельные, то для таких контуров применяют эквивалентные, или упрощенные, электросхемы. Каждую область цепи рассчитывают отдельно, а затем, представляя их вычисленными емкостями, объединяют в простую цепь.

Png?.png 600w, https://elquanta.ru/wp-content/uploads/2018/04/4-2-768×350..png 927w»>

Варианты получения эквивалентных схем

Особенности замены конденсаторов

К примеру, в наличии сеть переменного тока 12 В и две альтернативных группы последовательных конденсаторных элементов.

Конденсаторы подсоединяются в последовательный контур для увеличения напряжения, под которым они остаются работоспособными, но их общая емкость падает в соответствии с формулой для ее расчета.

Часто применяется смешанное соединение конденсаторов, чтобы создать нужную емкостную величину и увеличить напряжение, которое детали способны выдержать.

Можно привести вариант, как соединить несколько компонентов, чтобы выйти на нужные параметры. Если требуется конденсаторный элемент 80 мкФ при напряжении 50 В, но есть только конденсаторы 40 мкФ на 25 В, необходимо образовать следующую комбинацию:

  1. Два конденсатора 40 мкФ/25 В подсоединить последовательно, что позволит иметь в общей сложности 20 мкФ /50 В;
  2. Теперь вступает в действие параллельное включение конденсаторов. Пара конденсаторных групп, включенных последовательно, созданных на первом этапе, соединяются параллельно, получится 40 мкФ / 50 В;
  3. Две собранные в итоге группы соединить параллельно, в результате получим 80 мкФ/50 В.

Важно! Для того чтобы усилить конденсаторы по напряжению, возможно их объединить в последовательную электросхему. Увеличение общей емкостной величины достигается параллельным подключением.

Что необходимо учитывать при создании последовательной цепи:

  1. При соединениях конденсаторов оптимальный вариант – брать элементы с мало различающимися или с одинаковыми параметрами, вследствие большой разницы в напряжениях разряда;
  2. Для баланса токов утечки на каждый конденсаторный элемент (в параллель) включается уравнительное сопротивление.

Data-lazy-type=»image» data-src=»http://elquanta.ru/wp-content/uploads/2018/04/5-13-600×259.jpg?.jpg 600w, https://elquanta.ru/wp-content/uploads/2018/04/5-13-768×331..jpg 800w»>

Оцените статью:

Способы подключения конденсаторов в электрическую цепь

Схемы в электротехнике состоят из электрических элементов, в которых способы соединения конденсаторов могут быть разными. Надо понимать, как правильно подключить конденсатор. Отдельные участки цепи с подключенными конденсаторами можно заменить одним эквивалентным элементом. Он заменит ряд конденсаторов, но должно выполняться обязательное условие: когда напряжение, подводимое к обкладкам эквивалентного конденсатора, равняется напряжению на входе и выходе группы заменяющихся конденсаторов, тогда заряд емкости будет такой же, как и на группе емкостей. Для понимания вопроса, как подключить конденсатор в любой схеме, рассмотрим виды его включения.

Параллельное включение конденсаторов в цепь

Параллельное соединение конденсаторов — это когда все пластины подключаются к точкам включения цепи, образовывая батарею емкостей.

Параллельное соединение конденсаторов:

Параллельное соединение конденсаторов

Разность потенциалов на пластинах накопителей емкости будет одинаковая, так как они все заряжаются от одного источника тока. В этом случае каждый заряжающийся конденсатор имеет собственный заряд при одинаковой величине, подводимой к ним энергии.

Параллельные конденсаторы, общий параметр количества заряда полученной батареи накопителей, рассчитывается, как сумма всех зарядов, помещающихся на каждой емкости, потому что каждый заряд емкости не зависит от заряда другой емкости, входящей в группу конденсаторов, параллельно включенных в схему.

При параллельном соединении конденсаторов емкость равняется:

Формула и расшифровка

Из представленной формулы можно сделать вывод, что всю группу накопителей можно рассматривать как один равноценный им конденсатор.

Конденсаторы, соединенные параллельно, имеют напряжение:

Формула

Последовательное включение конденсаторов в цепь

Когда в схеме выполнено последовательное соединение конденсаторов, оно выглядит как цепочка емкостных накопителей, где пластина первого и последнего накопителя емкости (конденсатора) подключены к источнику тока.

Последовательное соединение конденсатора:

Формула

При последовательном соединении конденсаторов все устройства этого участка берут одинаковое количество электроэнергии, потому что в процессе участвует первая и последняя пластинка накопителей, а пластины 2, 3 и другие до N проходят зарядку посредством влияния. По этой причине заряд пластины 2 накопителя емкости равняется по значению заряду 1 пластины, но имеет обратный знак. Заряд пластины накопителя 3 равняется значению заряда пластины 2, но так же с обратным знаком, все последующие накопители имеет аналогичную систему заряда.

Формула нахождения заряда на конденсаторе, схема подключения конденсатора:

Последовательное соединение конденсаторов

Когда выполняется последовательное соединение конденсаторов, напряжение на каждом накопители емкости будет различное, так как в зарядке одинаковым количеством электрической энергии участвуют разные емкости. Зависимость емкости от напряжения такова: чем она меньше, тем большее напряжение необходимо подать на пластины накопителя для его зарядки. И обратная величина: чем выше емкость накопителя, тем меньше требуется напряжения для его зарядки. Можно сделать вывод, что емкость последовательно соединенных накопителей имеет значение для величины напряжения на пластинах — чем она меньше, тем больше напряжения требуется, а также накопители большой емкости требуют меньшего напряжения.

Основное отличие схемы последовательного соединения накопителей емкости в том, что электроэнергия протекает только в одном направлении, а это означает, что в каждом накопителе емкости составленной батареи ток будет одинаковым. В этом виде соединений конденсаторов обеспечивается равномерное накопление энергии независимо от емкости накопителей.

Группу накопителей емкости можно также на схеме рассматривать как эквивалентный накопитель, на пластины которого подается напряжение, определяемое формулой:

Основные моменты

Заряд общего (эквивалентного) накопителя группы емкостных накопителей последовательного соединения равен:

Формула

Общему значению емкости последовательно соединенных конденсаторов соответствует выражение:

Формула

Смешанное включение емкостных накопителей в схему

Параллельное и последовательное соединение конденсаторов на одном из участков цепи схемы называется специалистами смешанным соединением.

Участок цепи подсоединенных смешанным включением накопителей емкости:

Схема подключения конденсаторов

Смешанное соединение конденсаторов в схеме рассчитывается в определенном порядке, который можно представить следующим образом:

  • разбивается схема на простые для вычисления участки, это последовательное и параллельное соединение конденсаторов;
  • вычисляем эквивалентную емкость для группы конденсаторов, последовательно включенных на участке параллельного соединения;
  • проводим нахождение эквивалентной емкости на параллельном участке;
  • когда эквивалентные емкости накопителей определены, схему рекомендуется перерисовать;
  • рассчитывается емкость получившейся после последовательного включения эквивалентных накопителей электрической энергии.
Последовательное, параллельное и смешанное соединение конденсаторов

Накопители емкостей (двухполюсники) включены разными способами в цепь, это дает несколько преимуществ в решении электротехнических задач по сравнению с традиционными способами включения конденсаторов:

  1. Использование для подключения электрических двигателей и другого оборудования в цехах, в радиотехнических устройствах.
  2. Упрощение вычисления величин электросхемы. Монтаж выполняется отдельными участками.
  3. Технические свойства всех элементов не меняются, когда изменяется сила тока и магнитное поле, это применяется для включения разных накопителей. Характеризуется постоянной величиной емкости и напряжения, а заряд пропорционален потенциалу.

Вывод

Разного вида включения конденсаторов в цепь применяются для решения электротехнических задач, в частности, для получения полярных накопителей из нескольких неполярных двухполюсников. В этом случае решением будет соединение группы однополюсных накопителей емкости по встречно-параллельному способу (треугольником). В этой схеме минус соединяется с минусом, а плюс — с плюсом. Происходит увеличение емкости накопителя, и меняется работа двухполюсника.

Не отображаются имеющиеся вхождения: последовательное параллельное и смешанное соединение конденсаторов, последовательное и параллельное соединение конденсаторов, при параллельном соединении конденсаторов емкость.

Похожие статьи:

Последовательное соединение конденсаторов | Физика. Закон, формула, лекция, шпаргалка, шпора, доклад, ГДЗ, решебник, конспект, кратко

Раздел:

Конденсатор

При последовательном соединении кон­денсаторов соединяются между собою лишь две пластины разных конденсаторов. Если в каждом конденсаторе пластины обозначить буквами A и B, то при последовательном соединении пластина В1 будет соединена с пластиной A2, пластина B2 — с пластиной A3 и т. д. (рис. 4.77). Если цепочку по­следовательно соединенных конденсаторов соединить с источником тока, то обкладка A1 и обкладка Bn будут иметь одинаковые по значению заряды +Q и Q. Вследствие электростатической индукции все обкладки внутри цепочки будут иметь такие же, но попарно противоположные по знаку заря­ды, то есть все конденсаторы независимо от емкости будут иметь одинаковые заряды

Q1 = Q2 = Q3 = … = Qn = Q.

Рис. 4.77. Последовательное соединение конденсаторов

Вместе с тем общая разность потенциа­лов на концах цепочки будет равняться сум­ме разностей потенциалов на каждом кон­денсаторе:

Δφ = Δφ1 + Δφ2 + Δφ3 + … + Δφn.

Учитывая, что Δφ = Q / C, получим

Q / C = Q1 / C1 + Q2 / C2 + Q3 / C3 + … + Qn / Cn.

Разделим левую и правую части равенст­ва на Q:

1 / C = 1 / C1 + 1 / C2 + 1 / C3 + … + 1 / Cn.

При последовательном соединении конден­саторов обратное значение емкости цепочки равно сумме обратных значений емкостей каждого конденсатора.

Если последовательно соединить n оди­наковых конденсаторов, то общая электро­емкость будет в n раз меньше:

C = C0 / n. Материал с сайта http://worldofschool.ru

При последовательном соединении кон­денсаторов разной электроемкости C1, C2, C3, …, Cn общая электроемкость C будет меньше электроемкости наименьшего кон­денсатора:

Если C1 < C2 < C3 < … < Cn, то C < C1.

При последовательном соеди­нении конденсаторов уменьшается их емкость, но увеличивает­ся их рабочее напряжение.

На этой странице материал по темам:
  • Последовательное соединение конденсаторов доклад

  • Задача на электроемкость при последовательном соединении

  • Задачи с конденсаторами с решениями

  • Конденсаторы физика последовательно п

  • Последовательное соединение ел

Вопросы по этому материалу:
  • Какова общая электроемкость цепочки из последовательно соеди­ненных конденсаторов?

Вывод формулы последовательного соединения конденсаторов

Как правильно соединять конденсаторы?

У многих начинающих любителей электроники в процессе сборки самодельного устройства возникает вопрос: “Как правильно соединять конденсаторы?”

Казалось бы, зачем это надо, ведь если на принципиальной схеме указано, что в данном месте схемы должен быть установлен конденсатор на 47 микрофарад, значит, берём и ставим. Но, согласитесь, что в мастерской даже заядлого электронщика может не оказаться конденсатора с необходимым номиналом!

Похожая ситуация может возникнуть и при ремонте какого-либо прибора. Например, необходим электролитический конденсатор ёмкостью 1000 микрофарад, а под рукой лишь два-три на 470 микрофарад. Ставить 470 микрофарад, вместо положенных 1000? Нет, это допустимо не всегда. Так как же быть? Ехать на радиорынок за несколько десятков километров и покупать недостающую деталь?

Как выйти из сложившейся ситуации? Можно соединить несколько конденсаторов и в результате получить необходимую нам ёмкость. В электронике существует два способа соединения конденсаторов: параллельное и последовательное.

В реальности это выглядит так:


Параллельное соединение


Принципиальная схема параллельного соединения


Последовательное соединение


Принципиальная схема последовательного соединения

Также можно комбинировать параллельное и последовательное соединение. Но на практике вам вряд ли это пригодиться.

Как рассчитать общую ёмкость соединённых конденсаторов?

Помогут нам в этом несколько простых формул. Не сомневайтесь, если вы будете заниматься электроникой, то эти простые формулы рано или поздно вас выручат.

Общая ёмкость параллельно соединённых конденсаторов:

С1 – ёмкость первого;

С2 – ёмкость второго;

С3 – ёмкость третьего;

СN – ёмкость N-ого конденсатора;

Cобщ – суммарная ёмкость составного конденсатора.

Как видим, при параллельном соединении ёмкости нужно всего-навсего сложить!

Внимание! Все расчёты необходимо производить в одних единицах. Если выполняем расчёты в микрофарадах, то нужно указывать ёмкость C1, C2 в микрофарадах. Результат также получим в микрофарадах. Это правило стоит соблюдать, иначе ошибки не избежать!

Чтобы не допустить ошибку при переводе микрофарад в пикофарады, а нанофарад в микрофарады, необходимо знать сокращённую запись численных величин. Также в этом вам поможет таблица. В ней указаны приставки, используемые для краткой записи и множители, с помощью которых можно производить пересчёт. Подробнее об этом читайте здесь.

Ёмкость двух последовательно соединённых конденсаторов можно рассчитать по другой формуле. Она будет чуть сложнее:

Внимание! Данная формула справедлива только для двух конденсаторов! Если их больше, то потребуется другая формула. Она более запутанная, да и на деле не всегда пригождается .

Или то же самое, но более понятно:

Если вы проведёте несколько расчётов, то увидите, что при последовательном соединении результирующая ёмкость будет всегда меньше наименьшей, включённой в данную цепочку. Что это значить? А это значит, что если соединить последовательно конденсаторы ёмкостью 5, 100 и 35 пикофарад, то общая ёмкость будет меньше 5.

В том случае, если для последовательного соединения применены конденсаторы одинаковой ёмкости, эта громоздкая формула волшебным образом упрощается и принимает вид:

Здесь, вместо буквы M ставиться количество конденсаторов, а C1 – его ёмкость.

Стоит также запомнить простое правило:

При последовательном соединении двух конденсаторов с одинаковой ёмкостью результирующая ёмкость будет в два раза меньше ёмкости каждого из них.

Таким образом, если вы последовательно соедините два конденсатора, ёмкость каждого из которых 10 нанофарад, то в результате она составит 5 нанофарад.

Не будем пускать слов по ветру, а проверим конденсатор, замерив ёмкость, и на практике подтвердим правильность показанных здесь формул.

Возьмём два плёночных конденсатора. Один на 15 нанофарад (0,015 мкф.),а другой на 10 нанофарад (0,01 мкф.) Соединим их последовательно. Теперь возьмём мультиметр Victor VC9805+ и замерим суммарную ёмкость двух конденсаторов. Вот что мы получим (см. фото).


Замер ёмкости при последовательном соединении

Ёмкость составного конденсатора составила 6 нанофарад (0,006 мкф.)

А теперь проделаем то же самое, но для параллельного соединения. Проверим результат с помощью того же тестера (см. фото).


Измерение ёмкости при параллельном соединении

Как видим, при параллельном соединении ёмкость двух конденсаторов сложилась и составляет 25 нанофарад (0,025 мкф.).

Что ещё необходимо знать, чтобы правильно соединять конденсаторы?

Во-первых, не стоит забывать, что есть ещё один немаловажный параметр, как номинальное напряжение.

При последовательном соединении конденсаторов напряжение между ними распределяется обратно пропорционально их ёмкостям. Поэтому, есть смысл при последовательном соединении применять конденсаторы с номинальным напряжением равным тому, которое имеет конденсатор, взамен которого мы ставим составной.

Если же используются конденсаторы с одинаковой ёмкостью, то напряжение между ними разделится поровну.

Для электролитических конденсаторов.

При соединении электролитических конденсаторов (электролитов) строго соблюдайте полярность! При параллельном соединении всегда подключайте минусовой вывод одного конденсатора к минусовому выводу другого,а плюсовой вывод с плюсовым.


Параллельное соединение электролитов


Схема параллельного соединения

В последовательном соединении электролитов ситуация обратная. Необходимо подключать плюсовой вывод к минусовому. Получается что-то вроде последовательного соединения батареек.


Последовательное соединение электролитов


Схема последовательного соединения

Также не забывайте про номинальное напряжение. При параллельном соединении каждый из задействованных конденсаторов должен иметь то номинальное напряжение, как если бы мы ставили в схему один конденсатор. То есть если в схему нужно установить конденсатор с номинальным напряжением на 35 вольт и ёмкостью, например, 200 микрофарад, то взамен его можно параллельно соединить два конденсатора на 100 микрофарад и 35 вольт. Если хоть один из них будет иметь меньшее номинальное напряжение (например, 25 вольт), то он вскоре выйдет из строя.

Желательно, чтобы для составного конденсатора подбирались конденсаторы одного типа (плёночные, керамические, слюдяные, металлобумажные). Лучше всего будет, если они взяты из одной партии, так как в таком случае разброс параметров у них будет небольшой.

Конечно, возможно и смешанное (комбинированное) соединение, но в практике оно не применяется (я не видел ). Расчёт ёмкости при смешанном соединении обычно достаётся тем, кто решает задачи по физике или сдаёт экзамены 🙂

Тем же, кто не на шутку увлёкся электроникой непременно надо знать, как правильно соединять резисторы и рассчитывать их общее сопротивление!

Схемы в электротехнике состоят из электрических элементов, в которых способы соединения конденсаторов могут быть разными. Надо понимать, как правильно подключить конденсатор. Отдельные участки цепи с подключенными конденсаторами можно заменить одним эквивалентным элементом. Он заменит ряд конденсаторов, но должно выполняться обязательное условие: когда напряжение, подводимое к обкладкам эквивалентного конденсатора, равняется напряжению на входе и выходе группы заменяющихся конденсаторов, тогда заряд емкости будет такой же, как и на группе емкостей. Для понимания вопроса, как подключить конденсатор в любой схеме, рассмотрим виды его включения.

Параллельное включение конденсаторов в цепь

Параллельное соединение конденсаторов — это когда все пластины подключаются к точкам включения цепи, образовывая батарею емкостей.

Параллельное соединение конденсаторов:

Разность потенциалов на пластинах накопителей емкости будет одинаковая, так как они все заряжаются от одного источника тока. В этом случае каждый заряжающийся конденсатор имеет собственный заряд при одинаковой величине, подводимой к ним энергии.

Параллельные конденсаторы, общий параметр количества заряда полученной батареи накопителей, рассчитывается, как сумма всех зарядов, помещающихся на каждой емкости, потому что каждый заряд емкости не зависит от заряда другой емкости, входящей в группу конденсаторов, параллельно включенных в схему.

При параллельном соединении конденсаторов емкость равняется:

Из представленной формулы можно сделать вывод, что всю группу накопителей можно рассматривать как один равноценный им конденсатор.

Конденсаторы, соединенные параллельно, имеют напряжение:

Последовательное включение конденсаторов в цепь

Когда в схеме выполнено последовательное соединение конденсаторов, оно выглядит как цепочка емкостных накопителей, где пластина первого и последнего накопителя емкости (конденсатора) подключены к источнику тока.

Последовательное соединение конденсатора:

При последовательном соединении конденсаторов все устройства этого участка берут одинаковое количество электроэнергии, потому что в процессе участвует первая и последняя пластинка накопителей, а пластины 2, 3 и другие до N проходят зарядку посредством влияния. По этой причине заряд пластины 2 накопителя емкости равняется по значению заряду 1 пластины, но имеет обратный знак. Заряд пластины накопителя 3 равняется значению заряда пластины 2, но так же с обратным знаком, все последующие накопители имеет аналогичную систему заряда.

Формула нахождения заряда на конденсаторе, схема подключения конденсатора:

Когда выполняется последовательное соединение конденсаторов, напряжение на каждом накопители емкости будет различное, так как в зарядке одинаковым количеством электрической энергии участвуют разные емкости. Зависимость емкости от напряжения такова: чем она меньше, тем большее напряжение необходимо подать на пластины накопителя для его зарядки. И обратная величина: чем выше емкость накопителя, тем меньше требуется напряжения для его зарядки. Можно сделать вывод, что емкость последовательно соединенных накопителей имеет значение для величины напряжения на пластинах — чем она меньше, тем больше напряжения требуется, а также накопители большой емкости требуют меньшего напряжения.

Основное отличие схемы последовательного соединения накопителей емкости в том, что электроэнергия протекает только в одном направлении, а это означает, что в каждом накопителе емкости составленной батареи ток будет одинаковым. В этом виде соединений конденсаторов обеспечивается равномерное накопление энергии независимо от емкости накопителей.

Группу накопителей емкости можно также на схеме рассматривать как эквивалентный накопитель, на пластины которого подается напряжение, определяемое формулой:

Заряд общего (эквивалентного) накопителя группы емкостных накопителей последовательного соединения равен:

Общему значению емкости последовательно соединенных конденсаторов соответствует выражение:

Смешанное включение емкостных накопителей в схему

Параллельное и последовательное соединение конденсаторов на одном из участков цепи схемы называется специалистами смешанным соединением.

Участок цепи подсоединенных смешанным включением накопителей емкости:

Смешанное соединение конденсаторов в схеме рассчитывается в определенном порядке, который можно представить следующим образом:

  • разбивается схема на простые для вычисления участки, это последовательное и параллельное соединение конденсаторов;
  • вычисляем эквивалентную емкость для группы конденсаторов, последовательно включенных на участке параллельного соединения;
  • проводим нахождение эквивалентной емкости на параллельном участке;
  • когда эквивалентные емкости накопителей определены, схему рекомендуется перерисовать;
  • рассчитывается емкость получившейся после последовательного включения эквивалентных накопителей электрической энергии.

Накопители емкостей (двухполюсники) включены разными способами в цепь, это дает несколько преимуществ в решении электротехнических задач по сравнению с традиционными способами включения конденсаторов:

  1. Использование для подключения электрических двигателей и другого оборудования в цехах, в радиотехнических устройствах.
  2. Упрощение вычисления величин электросхемы. Монтаж выполняется отдельными участками.
  3. Технические свойства всех элементов не меняются, когда изменяется сила тока и магнитное поле, это применяется для включения разных накопителей. Характеризуется постоянной величиной емкости и напряжения, а заряд пропорционален потенциалу.

Вывод

Разного вида включения конденсаторов в цепь применяются для решения электротехнических задач, в частности, для получения полярных накопителей из нескольких неполярных двухполюсников. В этом случае решением будет соединение группы однополюсных накопителей емкости по встречно-параллельному способу (треугольником). В этой схеме минус соединяется с минусом, а плюс — с плюсом. Происходит увеличение емкости накопителя, и меняется работа двухполюсника.

Не отображаются имеющиеся вхождения: последовательное параллельное и смешанное соединение конденсаторов, последовательное и параллельное соединение конденсаторов, при параллельном соединении конденсаторов емкость.

При параллельном соединении конденсаторов к каждому кон­денсатору приложено одинаковое напряжениеU, а величина за­ряда на обкладках каждого конденсатора Q пропорциональна его емкости (рис. 2).

Общий заряд Q всех конденсаторов

Общая емкость С, или емкость батареи, параллельно включенных конденсаторов равна сумме емкостей этих конденсаторов.

Параллельное подключение конденсатора к группе других включенных конденсаторов увеличивает общую емкость батареи этих конденсаторов. Следовательно, параллельное соединение конденсаторов при­меняется для увеличения емкости.

4)Если параллельно включены т одинаковых конденсаторов ем­костью С´ каждый, то общая (эквивалентная) емкость батареи этих конденсаторов может быть определена выражением

Последовательное соединение конденсаторов

На обкладках последовательно соединенных конденсаторов, подключенных к источнику постоянного тока с напряжением U, появятся заряды одинаковые по величине с противоположными знаками.

Напряжение на конденсаторах распределяется обратно пропорционально емкостям конденса­торов:

Обратная величина общей емкости последовательно соединенных конденсаторов равна сумме обратных величин емкостей этих кон­денсаторов.

При последовательном включении двух конденсаторов их об­щая емкость определяется следующим выражением:

Если в цепь включены последовательно п одинаковых конден­саторов емкостью С каждый, то общая емкость этих конденса­торов:

Из (14) видно, что, чем больше конденсаторов п соединено последовательно, тем меньше будет их общая емкость С, т. е. по­следовательное включение конденсаторов приводит к уменьше­нию общей емкости батареи конденсаторов.

На практике может оказаться , что допустимое ра­бочее напряжение Up конденсатора меньше напряжения, на кото­рое необходимо подключить конденсатор. Если этот конденсатор подключить на такое напряжение, то он выйдет из строя, так как будет пробит диэлектрик. Если же последовательно включить не­сколько конденсаторов, то напряжение распределится между ними и на каждом конденсаторе напряжение окажется мень­ше его допустимого рабочего Up. Следовательно, последовательное соединение конденсаторов применяют для того, чтобы напряжение на каждом конденсаторе не превышало его рабочего напряжения Up.

Смешанное соединение конденсаторов

Смешанное соединение (последовательно-параллельное) кон­денсаторов применяют тогда, когда необходимо увеличить ем­кость и рабочее напряжение батареи конденсаторов.

Рассмотрим смешанное соединение конденсаторов на ниже­приведенных примерах.

где Q — заряд конденсатора или конденсаторов, к которым при­ложено напряжение U; С — электрическая емкость конденсатора или батареи соединенных конденсаторов, к которой приложено напряжение U.

Таким образом, конденсаторы служат для накопления и сохра­нения электрического поля и его энергии.

15.Дайте определение понятиям трех лучевая звезда и треугольник сопротивлений. Запишите формулы для преобразования трех лучевой звезды сопротивлений в треугольник сопротивлений и наоборот. Преобразуйте схему к двум узлам (Рисунок 5)

Рисунок 5- Схема электрическая

Для облегчения расчета составляется схема замещения электрической цепи, т. е. схема, отображающая свойства цепи при определенных условиях.

На схеме замещения изображают все элементы, влиянием которых на результат расчета нельзя пренебречь, и указывают также электрические соединения между ними, которые имеются в цепи.

1.Схемы замещения элементов электрических цепей

На расчетных схемах источник энергии можно представить ЭДС без внутреннего сопротивления, если это сопротивление мало по сравнению с сопротивлением приемника (рис. 3.13,6).

Приr= 0 внутреннее падение напряженияUо = 0, поэтому

напряжение на зажимах источника при любом токе равно

В некоторых случаях источник электрической энергии на расчетной схеме заменяют другой (эквивалентной) схемой (рис. 3.14, а), где вместо ЭДСЕ источник характеризуется его током короткого замыканияIK, а вместо внутреннего со­противления в расчет вводится внутренняя проводимостьg=1/r.

Возможность такой замены можно доказать, разделив равенство (3.1) на r:

где U/r = Io—некоторый ток, равный отношению напряжения на зажимах источника к внутреннему сопротивлению;E/r = IK — ток короткого замыкания источника;

Вводя новые обозначения, получим равенство IK= Io + I, которому удовлетворяет эквивалентная схема рис. 3.14,а.

В этом случае при любой величине напряжения на зажимах; источника его ток остается равным току короткого замыкания (рис. 3.14,6):

Источник с неизменным током, не зависящим от внешнего сопротивления, называют источником тока.

Один и тот же источник электрической энергии может быть заменен в расчетной схеме источником ЭДС или источником тока.

Последовательное соединение конденсаторов: формула :: SYL.ru

Под последовательным соединением подразумевают случаи, когда два или больше элемента имеют вид цепи, при этом каждый из них соединяется с другим только в одной точке. Зачем конденсаторы так размещаются? Как это правильно сделать? Что необходимо знать? Какие особенности последовательное соединение конденсаторов имеет на практике? Какая формула результата?

Что необходимо знать для правильного соединения?

Увы, но здесь не всё так легко сделать, как может показаться. Многие новички думают, что если на схематическом рисунке написано, что необходим элемент на 49 микрофарад, то достаточно его просто взять и установить (или заменить равнозначным). Но необходимые параметры подобрать сложно даже в профессиональной мастерской. И что делать, если нет нужных элементов? Допустим, есть такая ситуация: необходим конденсатор на 100 микрофарад, а есть несколько штук на 47. Поставить его не всегда можно. Ехать на радиорынок за одним конденсатором? Не обязательно. Достаточно будет соединить пару элементов. Существует два основных способа: последовательное и параллельное соединение конденсаторов. Вот о первом мы и поговорим. Но если говорить про последовательное соединение катушки и конденсатора, то тут особых проблем нет.

Зачем так делают?

Когда с ними проводятся такие манипуляции, то электрические заряды на обкладках отдельных элементов будут равны: КЕ=К123. КЕ – конечная емкость, К – пропускаемое значение конденсатора. Почему так? Когда заряды поступают от источника питания на внешние обкладки, то на внутренних может быть осуществлен перенос величины, которая является значением элемента с наименьшими параметрами. То есть если взять конденсатор на 3 мкФ, а после него подсоединить на 1 мкФ – то конечный результат будет 1 мкФ. Конечно, на первом можно будет наблюдать значение в 3 мкФ. Но второй элемент не сможет столько пропустить, и он будет срезать всё, что больше необходимого значения, оставляя большую емкость на первоначальном конденсаторе. Давайте рассмотрим, что нужно рассчитать, когда делается последовательное соединение конденсаторов. Формула:

Н=КЕ/ОЕК

  • ОЕ – общая емкость;
  • Н – напряжение;
  • КЕ – конечная емкость.

Что ещё необходимо знать, чтобы правильно соединить конденсаторы?

Для начала не забывайте, что кроме ёмкости они ещё обладают номинальным напряжением. Почему? Когда осуществляется последовательное соединение, то напряжение распределяется обратно пропорционально их ёмкостям между ними самими. Поэтому использовать такой подход имеет смысл только в тех случаях, когда любой конденсатор сможет предоставить минимально необходимые параметры работы. Если используются элементы, у которых одинаковая емкость, то напряжение между ними будет разделяться поровну. Также небольшое предостережение относительно электролитических конденсаторов: при работе с ними всегда внимательно контролируйте их полярность. Ибо при игнорировании этого фактора последовательное соединение конденсаторов может дать ряд нежелательных эффектов. И хорошо, если всё ограничится только пробоем данных элементов. Помните, что конденсаторы копят ток, и если что-то пойдёт не так, в зависимости от схемы может случиться прецедент, в результате которого из строя выйдут другие составляющие схемы.

Ток при последовательном соединении

Из-за того, что у него существует только один возможный путь протекания, он будет иметь одно значение для всех конденсаторов. При этом количество накопленного заряда везде обладает одинаковым значением. От емкости это не зависит. Посмотрите на любую схему последовательного соединения конденсаторов. Правая обкладка первого соединена с левой второго и так далее. Если используется больше 1 элемента, то часть из них будет изолированной от общей цепи. Таким образом, эффективная площадь обкладок становится меньшей и равняется параметрам самого маленького конденсатора. Какое физическое явление лежит в основе этого процесса? Дело в том, что как только конденсатор наполняется электрическим зарядом, то он перестаёт пропускать ток. И он тогда не может протекать по всей цепи. Остальные конденсаторы в таком случае тоже не смогут заряжаться.

Падение напряженности и общая емкость

Каждый элемент понемногу рассеивает напряжение. Учитывая, что емкость ему обратно пропорциональна, то чем она меньше, тем большим будет падение. Как уже упоминалось ранее, последовательно соединённые конденсаторы обладают одинаковым электрическим зарядом. Поэтому при делении всех выражений на общее значение можно получить уравнение, которое покажет всю емкость. В этом последовательное и параллельное соединение конденсаторов сильно разнятся.

Пример № 1

Давайте воспользуемся представленными в статье формулами и рассчитаем несколько практических задач. Итак, у нас есть три конденсатора. Их емкость составляет: С1 = 25 мкФ, С2 = 30 мкФ и С3 = 20 мкФ. Они соединены последовательно. Необходимо найти их общую емкость. Используем соответствующее уравнение 1/С: 1/С1 + 1/С2 + 1/С3 = 1/25 + 1/30 + 1/20 = 37/300. Переводим в микрофарады, и общая емкость конденсатора при последовательном соединении (а группа в данном случае считается как один элемент) составляет примерно 8,11 мкФ.

Пример № 2

Давайте, чтобы закрепить наработки, решим ещё одну задачу. Имеется 100 конденсаторов. Емкость каждого элемента составляет 2 мкФ. Необходимо определить их общую емкость. Нужно их количество умножить на характеристику: 100*2=200 мкФ. Итак, общая емкость конденсатора при последовательном соединении составляет 200 микрофарад. Как видите, ничего сложного.

Заключение

Итак, мы проработали теоретические аспекты, разобрали формулы и особенности правильного соединения конденсаторов (последовательно) и даже решили несколько задачек. Хочется напомнить, чтобы читатели не упускали из внимания влияние номинального напряжения. Также желательно, чтобы подбирались элементы одного типа (слюдяные, керамические, металлобумажные, плёночные). Тогда последовательное соединение конденсаторов сможет дать нам наибольший полезный эффект.

Последовательный и параллельный калькулятор емкости

[1] 2020/11/18 16:33 Моложе 20 лет / Другое / Очень /

Цель использования
Выяснение, какая комбинация конденсаторов у меня под рукой может создать значение, которое мне нужно в данной схеме
Комментарий / Запрос
Возможность добавления более двух конденсаторов

[2] 12.08.2020 18:32 Уровень 30 лет / Инженер / Полезный /

Назначение Используйте
Для проверки моей собственной работы по созданию проблем, которые должны решить младшие технические специалисты

[3] 2019/11/15 08:26 Уровень 20 лет / Средняя школа / Университет / аспирант / Полезно /

Цель использования
ДЛЯ ПОНИМАНИЯ
Комментарий / Запрос
ДЛЯ ПОЛУЧЕНИЯ ЗНАНИЙ

[4] 2019/04/10 06:25 Уровень 30 лет / Самозанятые люди / Очень /

Цель использования
Генератор Колпитца на УКВ, рассчитать общую емкость e на двойных варикапных диодах, используемых для настройки, а также общая емкость на делителе обратной связи.

[5] 2019/03/07 22:04 60 лет и старше / Пенсионер / Очень /

Цель использования
Помимо того, что я радиолюбитель, я также занимаюсь изготовлением кристаллических радиоприемников.
Для многих конструкций требуется воздушный конденсатор емкостью 500 пФ, но все, что я смог найти, это 630 пФ.
Итак, используя ваш калькулятор, я смог увидеть, сколько емкости мне нужно было добавить последовательно, чтобы снизить емкость конденсатора 630 пФ до 500 пФ.

Отлично сработало, мои искренние благодарности.

[6] 2018/08/27 12:07 Уровень 40 лет / Другое / Очень /

Цель использования
Отсутствие правильных значений на двух крышках.Используется для расчета заменяемых колпачков для старой магнитофонной деки.
Комментарий / запрос
Очень полезно

[7] 2018/08/17 04:15 Уровень 40 лет / Самостоятельно занятые люди / Очень /

Цель использования
Рассчитать шину питания для лампового усилителя

[8] 2018/08/11 16:16 60 лет и старше / Офисный работник / Государственный служащий / Полезно /

Цель использования
Устранение неисправностей источника питания. У меня был счетчик, который показывал максимум 10000 мкФ.Итак, мне пришлось последовательно соединить два одинаковых, чтобы проверить значение крышки фильтра.

[9] 2018/08/06 09:40 Уровень 20 лет / Инженер / Очень /

Цель использования
Расчет емкости для настройки антенны

[10] 13.06.2018 07 : 08 Уровень 50 лет / Средняя школа / Университет / Аспирант / Очень /

Цель использования
Два последовательно соединенных диода общей емкости для проектирования антенны.

Конденсаторы в схемах

  • Расчет схемы конденсатора
  • • Конденсаторы серии
  • • Конденсаторы, включенные параллельно

Конденсаторы последовательно.

Подобно резисторам и индукторам, конденсаторы могут быть подключены последовательно, параллельно или последовательно-параллельно. Последовательное размещение конденсаторов эффективно увеличивает толщину диэлектрика и, таким образом, снижает общую емкость. Поскольку общая емкость обратно пропорциональна расстоянию между пластинами, формула, которую мы используем для конденсаторов в СЕРИИ:

Обратите внимание, что напряжение на каждом конденсаторе будет обратно пропорционально емкости, при этом общее напряжение распределяется между конденсаторами, наименьшая емкость имеет наибольшее напряжение на ней, а наибольшая емкость имеет наименьшее напряжение и т. Д.

Конденсаторы параллельно.

Параллельное подключение конденсаторов эффективно увеличивает площадь пластин, поэтому общая емкость определяется суммой отдельных емкостей;

Все конденсаторы заряжаются до одинакового напряжения.

Обратите внимание, что при использовании этих формул значения должны вводиться в формулу в их БАЗОВЫХ ЕДИНИЦАХ, т.е. ФАРАДЫ (не мкФ), кулоны (не мкКл) и ВОЛЬТЫ (не мВ).

Кратковременно 47 нФ; (наноФарады) вводится как; 15 EXP-09 и 25mC (милликулонов) вводятся как; 25 ЕХР-03 и т. Д.Проще всего это сделать с помощью научного калькулятора.

Попробуйте вычислить общую емкость некоторых последовательных и параллельных цепей

Примеры последовательной и параллельной цепей.

Справка по математике

Нужна помощь с математикой электроники? Загрузите наш полезный буклет с советами по покупке и использованию научного калькулятора, а также по определению всех этих значений электроники. Неоценимая помощь в математике электроники в этом модуле!

Резистор, конденсатор и индуктор в последовательно-параллельном соединении

Резистор, конденсатор и индуктор в последовательном и параллельном соединении — формулы и уравнения

Следующие основные и полезные уравнения и формулы могут использоваться для проектирования, измерения, упрощения и анализа электрических цепей для различные компоненты и электрические элементы, такие как резисторы, конденсаторы и катушки индуктивности, соединенные последовательно и параллельно.

Сопротивление в последовательном и параллельном уравнениях

Сопротивление:

Общее эквивалентное сопротивление резисторов, подключенных последовательно или параллельно, определяется по следующей формуле:

Сопротивление последовательно:

Когда два или более двух резисторов подключены последовательно, как показано на рисунке, их эквивалентное сопротивление рассчитывается по формуле:

R Eq = R 1 + R 2 + R 3 +… R n

Сопротивление параллельно:

, когда резисторы в параллельной конфигурации, эквивалентное сопротивление становится:

Где

  • R Eq — эквивалентное сопротивление всех резисторов (R 1 , R 2 , R 3 … R n )

Похожие сообщения:

Дельта Δ до звезды Y (от Pi до Te e) Преобразование:

Межсоединение треугольником (Δ) также называется межсоединением Pi , а межсоединение звездой (Y) также обозначается как межсоединение Тройник (Т) .

От треугольника (Δ) до звезды (Y) Межсоединение:

От звезды (Y) до треугольника (Δ) Межсоединение

Для получения более подробной информации примеры, проверьте преобразование звезды в дельту и дельта в звезду.

Емкость в последовательном и параллельном уравнениях

Емкость:

Общая емкость конденсатора, подключенного параллельно и последовательно, приведена ниже:

Емкость последовательно:

Когда конденсаторы подключены последовательно В конфигурации эквивалентная емкость становится:

Параллельная емкость:

Емкость суммируется, когда они соединяются вместе в параллельной конфигурации

C Eq = C 1 + C 2 + C 3 +… C n

Где

  • C Eq — эквивалентная емкость всех конденсаторов (C 1 , C 2 , C 3 … C n )

Похожие сообщения:

Индуктивность в последовательном и параллельном уравнениях

Индуктивность:

9 0003 Расчет полной индуктивности катушек индуктивности внутри цепи аналогичен расчету резисторов.

Последовательная индуктивность:

Когда индукторы включены последовательно, как показано на рисунке, их индуктивности складываются.

L Eq = L 1 + L 2 + L 3 +… L n

Параллельная индуктивность:

Параллельная комбинация, эквивалентная катушки индуктивности задаются по формуле

Где

  • L Eq — эквивалентная индуктивность всех катушек индуктивности (L 1 , L 2 , L 3 … L n )

Связанные формулы и Посты с уравнениями:

конденсаторов, включенных последовательно и параллельно с их примерами

Существуют различные типы конденсаторов, в зависимости от области применения, которые подразделяются на разные типы.Подключение этих конденсаторов может осуществляться различными способами, которые используются в различных приложениях. Различные соединения конденсаторов работают как один конденсатор. Таким образом, общая емкость этого единственного конденсатора в основном зависит от того, как подключены отдельные конденсаторы. Таким образом, в основном есть два простых и распространенных типа соединений, такие как последовательное соединение и параллельное соединение. Используя эти соединения, можно рассчитать общую емкость. Есть некоторые соединения, которые также могут быть связаны с последовательными и параллельными комбинациями.В этой статье обсуждается обзор конденсаторов, включенных последовательно и параллельно, с их примерами.


Последовательные и параллельные конденсаторы

Конденсатор в основном используется для хранения электрической энергии, например, электростатической. Как только возникает потребность в увеличении количества энергии для накопления емкости, может потребоваться соответствующий конденсатор с увеличенной емкостью. Конструкция конденсатора может быть выполнена с использованием двух металлических пластин, которые соединены параллельно и разделены диэлектрической средой, такой как слюда, стекло, керамика и т. Д.

Диэлектрическая среда образует непроводящую среду между двумя пластинами и обладает исключительной способностью удерживать заряд.

Как только источник напряжения подключается к пластинам конденсатора, на одной пластине осаждается заряд + Ve, а на следующей пластине — заряд -Ve. Здесь общий накопленный заряд «q» может быть прямо пропорционален источнику напряжения «V».

q = CV

Где «C» — это емкость, и ее значение в основном зависит от физических размеров конденсатора.

C = εA / d

Где

‘ε’ = диэлектрическая проницаемость

‘A’ = площадь эффективной пластины

d = расстояние между двумя пластинами.

Если два или более конденсатора соединены последовательно, общая емкость этих конденсаторов мала по сравнению с емкостью отдельного конденсатора. Точно так же, когда конденсаторы подключаются параллельно, общая емкость конденсаторов является суммой емкостей отдельных конденсаторов.Используя это, выводятся выражения для полной емкости последовательно и параллельно. Также определены последовательные и параллельные части в комбинации конденсаторных соединений. И эффективная емкость может быть рассчитана последовательно и параллельно через отдельные емкости

Конденсаторы серии

Когда несколько конденсаторов соединены последовательно, напряжение, приложенное к конденсаторам, составляет «V». Если емкость конденсатора равна C1, C2… Cn, тогда соответствующая емкость конденсаторов при последовательном соединении будет «C».Приложенное напряжение на конденсаторах равно V1, V2, V3…. + Vn соответственно.

Конденсаторы серии

Таким образом, V = V1 + V2 + …… .. + Vn

Заряд, передаваемый от источника через эти конденсаторы, равен «Q», тогда

V = Q / C, V1 = Q / C1, V2 = Q / C2, V3 = Q / C3 и Vn = Q.Cn

Поскольку заряд, передаваемый в каждом конденсаторе, и ток во всей последовательной комбинации конденсаторов будут идентичными, это считается как «Q».

Теперь приведенное выше уравнение «V» можно записать следующим образом.

Q / C = Q / C1 + Q / C2 +… Q / Cn

Q [1 / C] = Q] 1 / C1 + 1 / C2 +… 1 / Cn]

1 / C = 1 / C1 + 1 / C2 + 1 / C3 +… 1 / Cn

Пример

Если конденсаторы подключаются последовательно, рассчитайте их емкость. Последовательное соединение конденсаторов показано ниже. Здесь последовательно подключено два конденсатора.

Конденсаторы в формуле последовательного включения: Ctotal = C1XC2 / C1 + C2

Значения двух конденсаторов: C1 = 5F и C2 = 10F

Cобщ. = 5FX10F / 5F + 10F

50F / 15F = 3.33F

Параллельные конденсаторы

Когда емкость конденсатора увеличивается, тогда конденсаторы подключаются параллельно, когда две связанные пластины соединяются вместе. Эффективная область перекрытия может быть добавлена ​​за счет стабильного расстояния между ними, и поэтому их одинаковое значение емкости превращается в удвоенную индивидуальную емкость. Конденсаторная батарея используется в различных отраслях промышленности, где конденсаторы используются параллельно. После того, как два конденсатора соединены параллельно, напряжение «V» на каждом конденсаторе будет одинаковым, то есть Veq = Va = Vb, а ток «ieq» можно разделить на два элемента, например «ia» и «ib».

Конденсаторы параллельно

i = dq / dt

Подставьте значение «q» в уравнение выше

= d (CV) / dt

i = C dV / dt + VdC / dt

Когда емкость конденсатора постоянна, тогда

i = C dV / dt

Применяя KCL к указанной выше схеме, уравнение будет

ieq = ia + ib

ieq = Ca dVa / dt + Cb dVb / dt

Veq = Va = Vb

ieq = Ca dVeq / dt + Cb dVeq / dt => (Ca + Cb) dVeq / dt

Наконец, мы можем получить следующее уравнение

ieq = Ceq dVeq / dt, здесь Ceq = Ca + Cb

Следовательно, если n конденсаторов соединены параллельно, равная емкость всего соединения может быть задана с помощью следующего уравнения, которое выглядит как соответствующее сопротивление резисторов при последовательном соединении.

Ceq = C1 + C2 + C3 +… + Cn

Пример

При параллельном подключении конденсаторов рассчитайте их емкость. Ниже показано параллельное соединение конденсаторов. Здесь конденсаторов, соединенных параллельно, два.

Конденсаторы в параллельной формуле: Ctotal = C1 + C2 + C3

Значения двух конденсаторов: C1 = 10F, C2 = 15F, C3 = 20F

Cобщ. = 10F + 15F + 20F = 45F

Падение напряжения на конденсаторах последовательно и параллельно будет изменяться в зависимости от индивидуальных значений емкости конденсаторов.

Примеры

Конденсаторы , включенные последовательно, и примеры , подключенные параллельно, обсуждаются ниже.

Конденсаторы в последовательном и параллельном примерах

Найдите значение емкости трех конденсаторов, включенных в следующую схему, со значениями C1 = 5 мкФ, C2 = 5 мкФ и C3 = 10 мкФ

Емкости конденсаторов: C1 = 5 мкФ, C2 = 5 мкФ и C3 = 10 мкФ

Следующая схема может быть построена с тремя конденсаторами, а именно C1, C2 и C3

Когда конденсаторы C1 и C2 соединены последовательно, емкость можно рассчитать как

1 / C = 1 / C1 + 1 / C2

1 / C = 1/5 + 1/5

1 / C = 2/5 => 5/2 = 2.5 мкФ

Если вышеуказанный конденсатор «C» можно подключить параллельно с конденсатором «C3», то емкость можно рассчитать как

C (Всего) = C + C3 = 2,5 + 10 = 12,5 мкФ

Следовательно, значение емкости можно рассчитать в зависимости от анализа последовательных и параллельных соединений в цепи. Это можно наблюдать, когда значение емкости уменьшается при последовательном включении. При параллельном подключении конденсатора значение емкости можно увеличить.Однако при расчете сопротивления все происходит наоборот.

Таким образом, это все о конденсаторах, включенных последовательно и параллельно, с примерами. Из приведенной выше информации, наконец, мы можем сделать вывод, что, используя последовательное и параллельное соединение конденсаторов, можно рассчитать емкость. Вот вам вопрос, а какая у конденсатора единица измерения?

конденсаторов последовательно и параллельно

Электростатический потенциал определяется как работа, выполняемая на единицу заряда.Более подробно, электрический потенциал в любой точке электрического поля можно определить как работу, совершаемую на единицу заряда при переносе единичного положительного заряда из бесконечности в эту точку, причем бесконечность является точкой вне электрического поля. Требуется работа, поскольку электрическое поле, производящее заряд, прикладывает силу к положительному заряду единицы после входа в поле. Емкость и электрический потенциал связаны таким образом, что они определяются как отношение электрического заряда (Q) к электрическому потенциалу (V), присутствующему в проводнике.

Формула для определения электростатического потенциала:

Внимание, читатель! Все, кто говорит, что программирование не для детей, просто еще не встретили подходящих наставников. Присоединяйтесь к демонстрационному классу для первого шага к курсу кодирования, специально разработан для учащихся 8-12 классов.

Студенты узнают больше о мире программирования в этих бесплатных классах , которые определенно помогут сделать правильный выбор карьеры в будущем.

В = Вт / кв (вольт)


Формула для емкости:

C = Q / В (Фарад)

Конденсаторы и емкость

Конденсаторы также называют конденсаторами. конденсатор, используемый для хранения электрического заряда, представляет собой не что иное, как проводники, расположенные на определенном расстоянии «d» параллельно друг другу, пространство между проводниками может быть либо вакуумом, либо изолирующим материалом / диэлектриком.Способность накапливать заряд в конденсаторе или конденсаторе известна как емкость. Несколько раз несколько конденсаторов подключаются в разных комбинациях для достижения разных уровней емкости. Эти комбинации могут быть произвольно сложными, но их можно разделить на два основных типа:

  1. Комбинация серий
  2. Параллельная комбинация

Комбинация серий

На приведенном ниже рисунке три конденсатора соединены последовательно с батареей. напряжения В.Обратите внимание, что на рисунке противоположные заряды одинаковой величины текут и накапливаются на пластинах конденсатора. Принцип сохранения заряда требует, чтобы заряд, который накапливается на пластинах конденсатора, был одинаковым по величине. Конечным результатом является комбинация, которая напоминает одиночный конденсатор с эффективным разделением пластин, которое больше, чем у отдельных конденсаторов, этот эквивалентный конденсатор показан на рисунке ниже. Большое разделение пластин означает меньшую емкость.

Соотношение емкости задается как,


C = Q / V

Его можно переписать как,

V = Q / C

Напряжения на отдельных конденсаторах будут,

В 1 = Q / C 1 , В 2 = Q / C 2 , В 3 = Q / C 3

Общее напряжение на всех конденсаторах будет,

V = V 1 + V 2 + V 3

, подставив выражения для отдельных напряжений,

V = Q / C 1 + Q / C 2 + Q / C 3

Пусть эквивалентная емкость равна C,

Q / C = Q / C 1 + Q / C 2 + Q / C 3

При упрощении приведенного выше уравнения соотношение становится,

Обычно для конденсаторов C 1 , C 2 , C 3 ,….

Параллельная комбинация

На приведенном ниже рисунке три конденсатора C 1 , C 2 , C 3 подключены параллельно источнику напряжения с потенциалом V. Получение эквивалентной емкости для этого случая относительно просто. Обратите внимание, что напряжение на каждом конденсаторе такое же, как у источника, поскольку он напрямую подключен к источнику. Таким образом, конденсаторы имеют такой же заряд, как и при индивидуальном подключении к источнику напряжения.Допустим, общий заряд конденсаторов равен Q,

Q = Q 1 + Q 2 + Q 3

Подставляем заряды с их соотношением с емкостями.


Q = Q 1 + Q 2 + Q 3

⇒ CV = C 1 V + C 2 V + C 3 V

⇒ C = C 1 + C 2 + C 3

Примеры проблем

Вопрос 1: Три конденсатора 3 пФ, 5 пФ , и 10 пФ подключены параллельно.Найдите эквивалентную емкость для системы.

Ответ:

Формула для последовательной емкости определяется как,

C = C 1 + C 2 + C 3

Дано: C 1 = 3pF, C 2 = 5 пФ и C 3 = 10 пФ

Подставляя эти значения в уравнение,

C = C 1 + C 2 + C 3

⇒ C = 3 + 5 + 10

⇒ C = 18pF

Вопрос 2: Три конденсатора 2pF, 2pF , и 4pF соединены последовательно.Найдите эквивалентную емкость для системы.

Ответ:

Формула для параллельной емкости определяется как,

Дано: C 1 = 2pF, C 2 = 2pF и C 3 = 4pF

, подставляя эти значения в уравнении


Вопрос 3: Найдите эквивалентную емкость для системы, показанной на рисунке ниже.

Ответ:

Формула для параллельной емкости определяется как,

, а формула для последовательной емкости определяется как,

C = C 1 + C 2 + C 3 +….

Это комбинация параллельной и последовательной емкостей.


Подставляя эти значения в уравнение,

C 1 = 10 мкФ, C 2 = 2,5 мкФ

C = C 1 + C 2

⇒ C = 10 + 2.5

⇒ C = 12,5

⇒ C = 0,29

Вопрос 4: Найдите эквивалентную емкость для системы, показанной на рисунке ниже.

Ответ:

Формула для параллельной емкости определяется как,

, а формула для последовательной емкости определяется как,

C = C 1 + C 2 + C 3 +….

Это комбинация параллельной и последовательной емкостей.

подставляя эти значения в уравнение,

C 1 = 100 мкФ, C 2 = 25 мкФ

C = C 1 + C 2


⇒ C = 100 + 25

⇒ C = 125

⇒ C = 2,9

Вопрос 5: Найдите эквивалентную емкость для системы, показанной на рисунке ниже.

Ответ:

Формула для параллельной емкости определяется как,

, а формула для последовательной емкости определяется как,

C = C 1 + C 2 + C 3 +….

Это комбинация параллельной и последовательной емкостей.

, подставляя эти значения в уравнение,

C 1 = 10 мкФ, C 2 = 0,3 мкФ

⇒ C = 0,29

C = + C 2

⇒ C = 0,29 + 2,5

⇒ C = 2,79


Калькулятор конденсаторов серии

• Калькуляторы электрических, радиочастотных и электронных устройств • Онлайн-преобразователи единиц

1 мФ = 0.001 F. 1 мкФ = 0,000001 = 10 F. 1 нФ = 0,000000001 = 10 F. 1 пФ = 0,000000000001 = 10 ² F.

Согласно второму правилу Кирхгофа, потенциал падает на В , В и В на каждом конденсаторе в группе из трех последовательно соединенных конденсаторов, как правило, различны, и полное падение потенциала В равно их сумме:

По определению емкости и из-за заряда Q группа конденсаторов, соединенных последовательно, является общей для всех конденсаторов, эквивалентная емкость C экв трех последовательно соединенных конденсаторов определяется как

или

Для группы из n подключенных конденсаторов последовательно эквивалентная емкость C eq является обратной величиной суммы обратных емкостей отдельных конденсаторов:

или

Эта формула для C экв используется для вычислений в этом калькуляторе.Например, три конденсатора, 10, 15 и 20 мкФ, соединенные последовательно, будут давать 4,62 мкФ:

Только для двух последовательных конденсаторов:

или

Если имеется n одинаковых конденсаторов C подключенных последовательно, тогда эквивалентная емкость будет

Обратите внимание, что формула для расчета общей емкости нескольких конденсаторов, соединенных последовательно, такая же, как и для расчета сопротивления группы резисторов, соединенных параллельно.

Также обратите внимание, что для группы, содержащей любое количество конденсаторов, соединенных последовательно, эквивалентная емкость всегда меньше, чем наименьший конденсатор в группе конденсаторов, и добавление любого нового конденсатора всегда будет уменьшать эквивалентную емкость группы.

Конденсаторы на печатной плате

Что касается падения напряжения на каждом конденсаторе, то это отдельная история. Даже если все конденсаторы, подключенные последовательно, равны, падение напряжения может быть различным, поскольку нельзя ожидать, что конденсаторы будут иметь одинаковую емкость и ток утечки.Конденсатор с наименьшей емкостью будет принимать наибольшее напряжение и, следовательно, будет самым слабым звеном в цепи.

Балансировочные резисторы уменьшают влияние вариации емкости

Чтобы помочь в равном распределении напряжения при последовательном подключении конденсаторов, к каждому конденсатору часто добавляются высокоэффективные балансировочные резисторы, чтобы гарантировать, что напряжение распределяется поровну. Резисторы действуют как делители напряжения и уменьшают влияние изменения емкости.Даже с этими резисторами лучше оставить значительный запас рабочего напряжения конденсаторов.

Если несколько конденсаторов подключены параллельно , разность потенциалов В, на конденсаторах одинакова и равна разности потенциалов между соединительными проводами. Общий заряд Q делится между конденсаторами, и если их емкости различаются, отдельные заряды Q₁ , Q₂ и Q₃ также будут разными.Общий заряд определяется как

Конденсаторы, подключенные параллельно

Учитывая, что по определению емкости эквивалентная емкость

мы получим

или

Для n конденсаторов параллельно,

То есть, если несколько конденсаторов подключены параллельно, их эквивалентная емкость определяется простым сложением емкостей всех конденсаторов в группе.

Как вы, наверное, заметили, конденсаторы ведут себя прямо противоположно резисторам: если резисторы подключены последовательно, их эквивалентные значения всегда будут выше, чем значения любых резисторов, подключенных последовательно, а их параллельное соединение приведет к уменьшению значений.

Конденсаторы на печатной плате

Эту статью написал Анатолий Золотков.

Электромагнетизм — Интуитивно, почему включение конденсаторов последовательно снижает эквивалентную емкость?

Может кто-нибудь объяснить, интуитивно .* = \ frac {U} {Q} $ вместо этого. (Действительно, сегодня это значение называется «эластичность». Спасибо Альфреду Центавра за ваш комментарий.)

Если бы они сделали это, «емкость» (которая была бы $ \ frac {U} {Q} $) последовательно соединенных конденсаторов увеличилась бы на , а не уменьшилась бы!

По этой причине я сомневаюсь, что можно объяснить это явление «интуитивно» без хотя бы со ссылкой на формулу $ C = \ frac {Q} {U} $.

Я также хотел бы дать общий ответ, который также применим к «настоящим» конденсаторам, не имеющим пластин…

… почему эквивалентная емкость последовательно соединенных конденсаторов меньше емкости любого отдельного конденсатора?

Сначала вы должны вспомнить, о чем вы говорите, если говорите о «емкости последовательного соединения»:

Вы, , — это , говоря о напряжении, измеренном на обоих концах последовательного соединения, и заряде, который протекал на одном конце последовательного соединения.

Вы , а не , говорите о напряжениях, измеренных внутри последовательного соединения и / или зарядов где-то внутри последовательного соединения.

Если несколько электронов втекают в один конец конденсатора или один конец последовательного соединения, такое же количество электронов вытечет из конденсатора или последовательного соединения на другом конце. Это количество электронов и есть «заряд конденсатора» $ Q $.

При последовательном соединении электроны, выходящие из первого конденсатора, перетекают во второй конденсатор. Это означает, что если некоторый заряд $ Q $ течет на один конец последовательного соединения, все конденсаторы будут заряжены с зарядом $ Q $.

Поскольку мы определили заряд, который поступал на один конец последовательного соединения, как «заряд последовательного соединения», «заряд последовательного соединения» составляет только $ Q $, а не $ N \ times Q $, если есть $ N $ конденсаторов последовательно, каждый из которых имеет заряд $ Q $.

С другой стороны, напряжение $ U $ описывает энергию, необходимую для переноса электрона из одной точки цепи в другую. Чтобы транспортировать электрон от одного конца последовательного соединения к другому, нам нужна энергия, чтобы транспортировать электрон от одного конца первого конденсатора к другому концу первого конденсатора.{*} = \ frac {U} {Q} $:

Поскольку напряжения суммируются, но заряд последовательного соединения равен заряду каждого отдельного конденсатора, емкость серии составляет:

$ \ displaystyle {C = \ frac {Q} {\ sum U_ \ text {конденсатор}}}

долл. США

Это означает, что числитель дроби $ \ frac {Q} {U} $ одинаков для одиночного конденсатора и последовательного соединения, но знаменатель больше при последовательном соединении.

.

Добавить комментарий

Ваш адрес email не будет опубликован. Обязательные поля помечены *